Этого треда уже нет.
Это копия, сохраненная 31 мая 2023 года.

Скачать тред: только с превью, с превью и прикрепленными файлами.
Второй вариант может долго скачиваться. Файлы будут только в живых или недавно утонувших тредах. Подробнее

Если вам полезен архив М.Двача, пожертвуйте на оплату сервера.
Тред тупых вопросов №167 728640 В конец треда | Веб
Тред вопросов о жизни, Вселенной и всём таком.

Спрашиваем то, за что в других местах выдают путёвку в биореактор. Здесь анонимные учёные мирового уровня критически рассмотрят любые гениальные идеи и нарисованные в Paint схемы.

Предыдущий тут: >>724642 (OP)
2 728646
На третьей ничего сложного. Сферическая система координат (сорт оф полярная), потом рандомные какие-то формулы из физики (наверн), сложение матриц, цилиндр какой-то из стереометрии - хз, дальше конус, тож какой-то бестолковый ну и немношк тригонометрии в конце, тож с физикой - томущо km в конце.
3 728647
>>728598 →
Из целой массы экспериментальных фактов.
68618412-1634274163.jpg40 Кб, 1280x956
4 728648
Раз уж о кварках заговорили, а объясните, откуда эта дурь взялась, что у элементарных частиц нет внутренней структуры? Раньше ж так же говорили об атомах, а там потом оказалось вон сколько внутри всего интересного.

пикрандом
5 728650
>>28648
Это не дурь, а экспериментальный факт. Никакой внутренней структуры у элементарных частиц не наблюдается. Собственно, они поэтому и называются элементарными.
6 728651
>>28645 (Del)
А как же сверхтекучесть и сверхпроводимость, постоянные магниты, фотохимия и лазер? Это только квантовые эффекты.
Поскольку макроскопические объекты состоят из большого числа структурных единиц, то и соответственно описание у них сложнее, чем простая волновая функция. То это не отменяет вышеперечисленные коллективные квантовые эффекты на больших масштабах.
7 728652
>>28645 (Del)

>Микроскопические быстрые тела - ???


В общем случае квантовая теория поля, которая в себя включает релятивистские описание электрона и фотонов, да и вообще любые поля.
на данный момент.PNG38 Кб, 1200x486
8 728658
>>28650

>Никакой внутренней структуры у элементарных частиц не наблюдается.


На данный момент?
9 728659
>>28651

>Поскольку макроскопические объекты состоят из большого числа структурных единиц


Это очень упрощенное видение на самом деле.
Чем к более мелкому масштабу мы обращаемся, тем более становится ясно, что эти самые структурные частицы представляют из себя осциляторы, на которые влияют соседние осциляторы да и вообще изменение внешних полей, особенно в случае резонансных эффектов.
10 728660
>>28658
Ну это и означает, что пока в эксперименте никакой внутренней структуры у них не наблюдается. Если увидят - значит, они не элементарные. Если они действительно элементарные, то ее никогда не увидят.
11 728661
>>28653 (Del)
Лазер невозможен в классической физике.
А вопрос туннелирование сводиться к степени изоляции соседа как системы.
12 728663
>>28660
Пока не увидели - значит попожже увидят.
Как вообще у чего-то без внутренней структуры могут быть свойства?
13 728666
>>28663
Может структура это и есть это самое свойство
14 728669
>>28666
Нет, это тогда уже какой-то идеализм. Свойства определяются начинкой, а не наоборот. Начинка - первична, свойства - вторичны. Без начинки не может быть свойств. Ну бля, нет разве?
15 728670
>>28645 (Del)

>Микроскопические быстрые тела


Это какие?
16 728671
>>28648
Струны?А потом будут ломать голову,а струны из чего состоят
17 728674
>>28669
Ну тогда возникает вопрос что придает свойства этой самой начинке. Еще более мелкая начинка? И где тогда предел дробимости? Вобщем все равно уход философский треп.
18 728678
>>28671
А что с ними нет так? Струны по сути те же самые частицы только в другом представление.
19 728683
>>28674

>предел дробимости?


Нет предела дробимости, например. Не?

>философский треп.


Рано или поздно оно туда и уйдет, да.
20 728689
>>28686 (Del)
В КТП не рассматривает гравитационные эффекты даже на совсем ультрарелятивистских частиц. В любом случае метод возмущений уже становиться не применим на энергиях на порядки меньше, чем самогравитация станет важна. Да и скорее полезут дополнительные размерности или суперсимметрия, чем гравитационное говно.
21 728694
Почему при возвращении первых ступеней ракет в спейсикс не используют парашюты?

Как я понимаю если садить ступень чисто на парашюте, то мягко посадить не получится, и там все погнется и поломается, что дальше уже будет не пригодно.

Но почему бы не использовать парашют как промежуточный этап для замедления. Ступень отделяется, потом двигателями разворачивается в нужное положение - долетает до слоев атмосферы, где уже можно использовать парашюты, опять же двигателем притормаживается до скорости, чтоб парашюты не оторвало - открывается парашюты и дальше спускаются на них - за условные 1000 метров до поверхности парашюты срезаются, и сама посадка уже двигателями. В теории кажется что так можно сэкономить много топлива, необходимо для снижения скорости и посадки ступени.

Еще идея с парашютами для посадки не всей ступени, а только двигателей (которые типо самая дорогая и ценная часть) - посадка чисто на парашютах, а для смягчения перед столкновением с землей выстреливается нечто вроде подушки безопасности, чтоб он ударился не об твердую поверхность, а упал в эту "подушку".
22 728698
>>28645 (Del)

> время твоей жизни не определяется частотой событий измерения твоего состояния


ну если учитывать что измерения зачастую коллапсируют волновую функцию то это такой себе контрпример
23 728699
>>28694

>спейсикс


Ты промазал. Спейсикс там: >>728116 (OP)
24 728700
>>28699
Вопрос не про спейсикс, а в принципе про технологии возвращения ступеней
25 728701
>>28700
Просто сейчас возвращают вроде только они. Вот и вопрос - почему решили именно таким образом, который с дивана выглядит каким-то очень неэффективным. Ты тратишь топливо чтоб поднять ракету, а потом чуть ли не столько же, чтоб спустить ее обратно. У них масса выводимых грузов чуть не в 2 раза падает, если они ступени возвращают. Вот и непонятно, почему другие варианты не применимы, а только посадка двигателем на всем промежутке падения.
26 728702
>>28701

>а потом чуть ли не столько же


Явно гораздо меньше, ибо к моменту торможения ракета уже избавилась от второй ступени и части топлива то есть значительно полегчала.

>а только посадка двигателем на всем промежутке падения.


Двигатели работают только при торможении и непосредственно посадке, остальную часть пути ракета изображает собой утюг

>почему другие варианты не применимы


Парашютная посадка неуправляема и ступень бы побилась при посадке, Прикручивать крылья - масса возрастет весьма значительно и конструкция сильно усложнится.
27 728703
>>28701

>У них масса выводимых грузов чуть не в 2 раза падает


15 тонн против 22 это не в 2 раза.
28 728716
>>28663

>Пока не увидели - значит попожже увидят.


Не значит. Электрон известен с конца XIX в., и никаких даже намеков на внутреннюю структуру у него до сих пор не найдено.

>Как вообще у чего-то без внутренней структуры могут быть свойства?


Очень просто. Элементарные частицы описываются простым набором квантовых чисел: масса, заряд, спин, время жизни и т.п. Никакая внутренняя структура здесь не требуется.
29 728724
>>28640 (OP)
Я всю жизнь считал, что чем больше человек летает и путешествует, тем он больше синхронизируется с полем Земли и меньше стареет, а тот, кто дома сидит, его гравитация постоянно давит и из-за того, что скорость равна нулю стареет быстрее
в идеале надо по земле вращаться со скоростью вращения земли, тогда будет похуй на время
30 728727
>>28724
В более сильном гравитационном поле время течет медленнее, чем в менее сильном. Поэтому медленнее стареет тот, кто находится на поверхности Земли, а тот, кто, соответственно, летает в космосе - быстрее. Это эффект ОТО. Хотя разница там мизерная, т.к. гравитация у Земли очень слабая для ОТО. Вращение тут ни при чем.
31 728728
Что показывает и для чего формула E=mc^2?
32 728729
>>28727
Значит так, в общем есть корреляция
33 728731
>>28728
Энергию покоя. Или мощность взрыва от аннигиляции.
34 728732
>>28728
Энергия покоя.
Нужна для того, чтобы определить действие для релятивистского движения.
Показывает какое действие совершает масса при движение в пространстве-времени.
35 728734
>>28732
А масса при достижении скорости света станет больше?
36 728735
Есть у ОТО конкурент?
37 728736
>>28734
Нет.
Масса инвариант, и в пространстве-времени она и так движется со скоростью света.
38 728737
Где доказательство,что массивные тела искривляют пространство и поэтому тело падает на массивный объект?Я же не улетаю в космос при прыжке.
Гипотеза с гравитоном выглядит более логичным объяснением притяжения
39 728742
>>28735
конкурент в чем? в своих рамках применимости (в дырки не лезть) ТО не вызывает нареканий, а нынешние кандидаты в "теории всего": что петли, что струны - ещё довольно сырые чтобы с чем-то конкурировать
40 728752
>>28702

>>Двигатели работают только при торможении и непосредственно посадке, остальную часть пути ракета изображает собой утюг



Это не принципиально когда именно они включаются, в любом случае придется погасить одну и ту же скорость. Ступень падает с ускорением 10 м/с и при падении к примеру с 50км наберет скорость от 0 до 1000 м/с. И эту тысячу нужно будет погасить, можно это сделать одним заходом перед посадкой или постепенно, не давая слишком сильно разгоняться при падении.

В теории кажется что парашютом можно погасить большую часть этой скорости, а потом включением двигателей сделать мягкую посадку.

>> Явно гораздо меньше, ибо к моменту торможения ракета уже избавилась от второй ступени и части топлива то есть значительно полегчала.



Это топливо поднимается на высоту, утяжеляя стартовую массу ракеты, это как-то выглядит очень не эффективным. На Земле все таки есть атмосфера, которую можно как-то использовать для посадки (парашюты, планеры), и полностью это игнорировать выглядит как-то странно, что это может быть рациональным и экономически обоснованным.
41 728753
>>28737
Это математическая теория, с помощью которой можно рассчитывать разные гравитационные штуки. В реальности пространство разумеется никуда не искривляется. Это просто абстракция. Вариант с гравитацией как силой - тоже абстракция. Есть наблюдаемое явление (тела притягиваются), ты придумываешь какую-то абстракцию-теорию, которой можно объяснить и предсказать движение этих тел - если все получается, то эту теорию используют. А как там на самом деле работает механизм притяжения - никто не знает.
42 728764
>>28757 (Del)

>Кек, пук, я так на умных сайтах прочитал в статейках от умных людей, кек, пук.

43 728768
>>28764
Ок, я тупой, ты умный. Теперь расскажешь откуда ты знаешь, что "В реальности пространство разумеется никуда не искривляется"?
44 728770
>>28768
Я не он, но экспериментов, достоверно подтверждающих явление искривления пространства современной наукой не проведено.
45 728775
>>28728
Эквивалентность массы и энергии - одно может перейти в другое.
46 728776
>>28737

>Где доказательство,что массивные тела искривляют пространство


Гравитационное линзирование.

>Гипотеза с гравитоном выглядит более логичным объяснением притяжения


Ты эти гравитоны уже обнаружил в эксперименте?
47 728778
>>28776

>Гравитационное линзирование.


Это может быть влияние гравитонов
48 728779
>>28752

>в любом случае придется погасить одну и ту же скорость


Но поскольку массы уже гораздо меньше, то на погашении этой скорости уйдет меньше топлива чем на ее набор

>кажется что парашютом можно погасить большую часть этой скорости, а потом включением двигателей сделать мягкую посадку.


Космическая скорость гасится торможением двигателем в верхних слоях атмосферы когда парашют бесполезен. При посадке у ракеты скорость утюга, и это скорость гасится уже с меньшими усилиями.

> полностью это игнорировать выглядит как-то странно, что это может быть рациональным и экономически обоснованным.


Я думаю что они там все-таки подсчитали на салфетке что к чему.
49 728784
>>28778
Каких гравитонов? Они с тобой в комнате?
50 728786
>>28779

>> Я думаю что они там все-таки подсчитали на салфетке что к чему.



Да это понятно конечно, что они посчитали, не понятно, как они посчитали)
Идея переиспользования деталей ведь давно уже есть. Те же американцы вбухали миллиарды в шаттлы, которые не взлетели, в Союзе вбухали свои миллиарды в Буран, который тоже не взлетел. Капсулы с космонавтами садят на поверхность с помощью парашютов все годы, сколько летают. И тут приходит Маск и говорит, а давайте садить детали ракеты точно так же, как они и взлетают. И типо оказалось это выгодно, что сейчас другие начинают проекты с похожей реализацией. Почему до этого раньше не додумались? Технологически ведь ничего особо не изменилось, те же жрд двигатели, те же ступени ракет, разве что может электроника управляющая стала более точной. Но эту идею видимо раньше даже не рассматривали или сразу отвергали, а теперь она почему-то стала главной.
51 728787
>>28786
Скорее всего все-таки энерция мышления: как это садит такую ебаку так как она взлетает? С Маском сошлось наверно потому что и энтузиаст и миллиардер - сказал "хочу вот эдак" и денег дал.
52 728799
>>28770
а как тогда гравиволны ловят?
53 728805
>>28786
идеи рассматривали ещё во время первых полётов, но тогда все пришли к очевидно более простым в реализации парашютам, на разработку реюза как удешевления никто денег не давал, да и целесообразность что-то там экономить не такая самоочевидная как сейчас кажется
парашуты в начале рывка дают много нагрузки на растяжение, а при реактивном способе - более равномерную и на такое же продольное сжатие как и при взлёте

технологическое совершенство явно с тех пор скакнуло - жыпиэсы, устойчивая к помехам от факела электроника и вообще много всего
ещё немаловажный момент политическая воля смотреть как взрываются ракеты под смехуечки про усталость металла
54 728807
>>28770
Проведено дохуя экспериментов, подтверждающих все предсказания ОТО. Если ты предложишь теорию, лучшую, чем ОТО, без искривления пространства-времени, и она подтвердится, то пожалуйста. А пока проследуй нахуй.
55 728811
>>28809 (Del)
Которую выкинули на помойку в начале двадцатого века?
56 728814
>>28809 (Del)
А еще есть торсионные поля, волновой геном и прочие охуительные истории. Прими таблетки.
57 728816
>>28807

>>Проведено дохуя экспериментов, подтверждающих все предсказания ОТО



Кстати, а как насчет темной материи? Я не очень в теме и лень сейчас гуглить, поэтому возможно ошибаюсь. Но как помню, там звезды в галактиках разбегались или закручивались по наблюдениям не с теми скоростями, которые предсказывала теория. И что в итоге.. теория не может ошибаться, поэтому введен невидимый и ненаблюдаемый костыль "темная материя", который нужен чисто для того, чтоб подогнать расчеты к наблюдаемым явлениям.
58 728818
>>28816

>Я не очень в теме


>ряя колдуны-ученые предумоли темную материю эта фсе вранье вы фсе врети я скозал!


Классика антисетипетушиного жанра.
59 728819
>>28818
Ну и в чем он не прав?
60 728820
>>28819
В том, что тупой необразованный хуесос, которому хватает тупости выносить суждения на темы космического масштаба, в которых он нихуя не понимает и сам же это признает, космической же глупости.
61 728821
>>28807

>> без искривления пространства-времен



Искривление пространства для меня это какое-то изнасилование логики. Я не верю, что это кто-то может понимать.

Пространство это нечто базовое, основное, эталонное, какая-то элементарная логическая единица, от которой выстраивается все остальное. Эта не физическая реальность, это нечто относящееся к восприятию человеком физической реальности.

Чтобы что-то было было искривлено, значит что-то должно быть прямолинейно. Что является эталоном, понятием этой прямолинейности? Выбирается некое понятие, логическая сущность, которая принимается как прямолинейное, и тогда все другое может быть искривлено относительно этого эталона. Если же можно искривлять сам эталон, то разрушается само понятие прямолинейности-искривленности.

Хз как это описать и передать, я не могу осознать это этого "искривленного пространства" и не верю, что кто-то может. Все рисунки объяснения ОТО - это рисунки не искривленного пространства, а кривых линий, какой-то криволинейной системы координат в неперывном и прямолинейном пространстве. Я потому и могу воспринимать эту искривленную систему координат, потому что у меня есть понятие прямолинейного пространства.

Еще у меня есть такая метафора. Вот есть "прямая", есть "кривая", а если ввести понятие "искривленная прямая", которая не является ни "прямой", ни "кривой", а какая-то отдельная сущность, или подвид/общий случай "прямой", но при этом не являясь "кривой".. Это все невозможно представить, и это просто какое-то извращение над логикой. Могут быть только две логические сущности - прямая и кривая, искривленная прямая - это какая-то бессмысленная ненужная хуйня.

Это все не к тому, что математическй аппарат какой-то теории (расчет/предсказание экспериментов и наблюдаемых явлений) не верен, а к тому что буквальная интерпретация и натягивание математического аппарата физической теории на физическую реальность может давать ссовершенно ебанутые и нелогичные результаты.
62 728822
>>28821
ебать, и нахуя простыню высрал о том что ты интеллектуально немощный и не можешь понять основополагающих вещей в современной науке? и можешь только вижать "врети, нипанимаю, нелогично"
ты и так достаточно ясно выразил свою позицию до этого
63 728826
>>28820
>>28822
Таблетки прими, маня буйная

Этот тред называется "тред тупых вопросов", я пишу здесь о том, что для меня не понятно, чтоб возможно какой-нибудь адекватный анон мне адекватно ответил
64 728829
>>28826

> Этот тред называется "тред тупых вопросов", я пишу здесь о том, что для меня не понятно


> это какое-то изнасилование логики. Я не верю



вот именно что это тред вопросов а не высеров про веру и восприятие тобой твоих личных маняпредставлений про "логические сущности"
65 728832
>>28829
Это вопрос про понимание искривления пространства в форме "Чем криволинейное движение в прямолинейной системе координат отличается от прямолинейного движения в искривленной системе координат на уровне физического мира, а не математических расчетов". На самом деле не думаю, я единственный кто это не понимают, скорее большинство/никто, иначе не было бы миллионов видео на ютюбах с попыткой объяснить необъясняемое
66 728834
>>28826
Чушка, ты не задала ни одного вопроса, а сходу начала высирать категоричные суждения по теме, в которой ты нихуя не понимаешь, в чем ты сама же и призналась. Т.е. ты просто пришла и на виду у всех насрала себе полные штаны.
67 728836
>>28832
Учи математику ОТО. Потом поймешь, что такое метрика и ее искривление. Так ты себе только голову забиваешь хуйней.
Про то, как выглядит искривление пространства в реальности, говорилось выше. Эффект гравитационного линзирования.
68 728839
>>28816

>Я не очень в теме и лень сейчас гуглить


Ну так погугли блядь. Скорости орбит звёзд в галактиках далеко не единственное свидетельство существования тёмной материи. Это во-первых. Во-вторых, изначально под тёмной материей подразумевалось вообще любое вещество, которое мы не видим, например рандомно летающие планеты и астероиды, чёрные дыры и т.п. Сейчас уже известно, что такие объекты (MACHO их ещё называют) не могут отвечать за большую часть тёмной материи.

>невидимый и ненаблюдаемый


Настолько же наблюдаемый, насколько и атомы на момент их открытия например. Или ядра атомов, или электроны.
69 728851
>>28836

>> Про то, как выглядит искривление пространства в реальности, говорилось выше. Эффект гравитационного линзирования.



Гравитацио́нная ли́нза — массивное тело (планета, звезда, галактика, скопление галактик, скопление тёмной материи), изменяющее своим гравитационным полем направление распространения электромагнитного излучения, подобно тому как обычная линза изменяет направление светового луча.

Так это ведь не про искривление пространства, а про влияние массивных тел на распространение электромагнитного излучения. Они написали про обычную линзу, они ведь не пишут, что линза искривляет пространство, а когда в место линзы появляется массивное тело, то оно почему-то его искривляет.
70 728852
>>28839

>> Скорости орбит звёзд в галактиках далеко не единственное свидетельство существования тёмной материи.



Я не понимаю, как это работает. Ты рассчитываешь скорости орбит звезд. Смотришь в телескоп и видишь, что скорости отличаются от рассчитанных. Почему отсюда следует вывод, что там есть что-то, чего мы не видим, а не вывод - мы неправильно посчитали?
71 728853
>>28836

>> Учи математику ОТО


Я про это и говорю, что ОТО (и вся физика современная) это про математику. И пытаться эту математику натягивать на физический мир, пытаясь понять и объяснить в привычных человеку понятиях - занятие бессмысленное
72 728854
>>28839

>> Настолько же наблюдаемый, насколько и атомы на момент их открытия например. Или ядра атомов, или электроны.



Так они ведь до сих пор ненаблюдаемы. Разве можно увидеть электрон или ядро атома? Ты подаешь электричество на одном конце установки - и смотришь как что-то почему-то мигнуло/затрещало. Что там произошло между этими событиями - черный ящик. Ты придумываешь теорию, которая поможет тебе предсказать, что случится, если например поставить посередине установки пластину с двумя щелями, если получилось - хорошо, твою теорию используют. Но ты все равно не можешь знать что там происходит на самом деле.

В этом смысле без разницы на темную материю, есть она, нет ее, если этот костыль для сходимости расчетов с наблюдениями работает, то это тоже теория, которая типо работает. Проблемы могут начинаться, что для подгонки фактов к теории сначала ты придумываешь темную материю, потом появляется темная энергия, ну а потом следует ожидать и черного властелина
sage 73 728855
>>28640 (OP)
надо бате о дваче рассказать, а то он заебал меня уже своей антинаучной ахинеей.
74 728856
можете наблюдать класические визги антисетипетуха на тему "научной неорелигии"
75 728871
буп
76 728885
>>28852

>а не вывод - мы неправильно посчитали?


Потому-что скорости рассчитываются не с потолка а по формулам которые очень хорошо работают во всех других случаях.
77 728889
>>28888 (Del)
Все записано в толстых талмудах обтянутых человеческой кожей которые хранятся в подвалах тайных астрономических обществ.
Вроде по эффекту Доплера
78 728891
>>28889
То есть измеряются по по эффекту Доплера а расчитываются по другим тайным талмудам. По теории относительности
79 728893
>>28888 (Del)
Используют теорему о вириале.
Зная потенциал, можно вычислить среднею скорость финитного движения(элементов связанных в систему).
Потенциал вычисляет по распределение светящейся материи (газ, пыль и звезды).
80 728894
>>28891
Считают по ньютоновской гравитации или его приближении - гравитомагнетизм. Использовать ТО нет смысла тут.
81 728908
>>28905 (Del)

> несветящейся материи


вот эта хуйня и называется волшебным словом "тёмная"
не светится = не взаимодействует (или почти никогда не взаимодействует) с обычной материей
82 728915
>>28904 (Del)
По изменению света самой звезды.
83 728916
>>28811
Ну рановато выкинули. На ней же все строили, всю физику. Эфир это база физики, основа основ. Если его выкинуть то получается хуета вместо физики.
84 728920
>>28916
Выкинули когда поняли что его не существует. Это как ни странно была вполне себе научная теория от которой отказались когда она не подтвердилась.
85 728921
>>28918 (Del)
Красный - летит к нам всем писец, Синий - улетает от нас можно выдыхать
86 728922
>>28921
Или наоборот.
87 728924
>>28920

>Выкинули когда поняли что его не существует.


Нет. Ты уверовал в подмену понятий. Эксперимент показал "отсутствие" эфирного ветра, а не отсутствие эфира. Причина такого результата, как выяснилось, в несовершенстве теории и конструкции интерферометра. То есть ожидали по теории что интерферометр данной конструкции может засечь ветер, но реальность как всегда преподносит коррективы - такой интерферометр обладает крайне низкой, на уровне шумов, чувствительностью в воздухе и совсем теряет чувствительность в вакууме. Интерферометр другой конструкции спокойно детектит ветер.
88 728958
Почему всё, относящееся ко второй планете - венерианское, а болезни - венерические?
89 728964
>>28924

>Интерферометр другой конструкции спокойно детектит ветер.


В студию его!
fieldimage959665.jpg214 Кб, 1014x900
91 728971
Поясните, можно ли радиотелескопом найти экзопланеты? Или они только для изучения звёзд?
92 728972
>>28916

>Эфир это база физики, основа основ.


школохуета, ты не заебалась так толсто набрасывать?
93 728973
>>28967
Сейчас бы ссылаться на уволенного фрика.
94 728978
>>28924

>Нет. Ты уверовал в подмену понятий. Эксперимент показал "отсутствие" эфирного ветра, а не отсутствие эфира. Причина такого результата, как выяснилось, в несовершенстве теории и конструкции интерферометра.


Да просто пидары с какого-то хрена выдумали, что у эфира на поверхности Земли должен быть ветер - ну и естественно не нашли его. Поэтому пидары заключили, что раз эфирного ветра на поверхности Земли нет - то и эфира нет.
Это всё равно что сказать "солнце светит из-за макаронного монстра - а раз монстра никто найти не может, то и солнца нет!" )))
На самом деле Земля уносится потоком плотноприжатых сфер вокруг Солнца. Сферы, которые ближе к Солнцу делают оборот за меньшее время - поэтому возникает разность скоростей между сферами, которые давят на Землю. Сферы движутся быстрее Земли, натыкаются на Землю - в сторону Солнца уйти не могут, там скорости выше, уходят в сторону от Солнца, а там скорости ниже. Натыкаются на плотноприжатые и медленные - поэтому огибают Землю - так образуется вихрь вокруг Земли из плотноприжатых сфер, та самая гравитационная воронка (искривление "пространства"). Но этот вихрь не уловить на поверхности - ибо на поверхности Земли эфир замедляет атмосфера. На орбите тоже не уловить - потому что искусственные спутники чтобы не упасть набирают скорость эфирного вихря. Нужны два пениса Безоса с разносом - и вот когда они поднимутся до 100км - то между двумя кораблями ставить опыты.
95 729005
>>28978

>плотноприжатых сфер


Что это вообще? Ты хоть ссылку скинь почитать - нихуя не гуглится.
96 729007
>>29005
ссылку кидать куда? в эту клоаку, где трут мои посты? да нахой мне надо так унижаться - я просто мимо проходил... смотрю, что оп-ходячаяфлешка запостила моих картиночек в шапке - решил зайти - но как тред был говном в одну харю, так и остался. пусть этот шиз дальше общается сам с собой - сам себе вопросы задает, сам отвечает...
97 729008
>>29007
Не понял, что за флешка и кто там у тебя что трет.
98 729010
Кэфиршики опять сюда протекают.
Не хватает только варпоебов на квантовых свехсветовых пузырях.
Чому всякие макси не запилят пусковую петлю или хотя бы в Сахаре не запилят гигантскую солнечную станцию, которая будет из песка кремний добывать ?
99 729014
>>29010
Кефирщики дуются на оп-а за то, что тот не делится пряниками. Оп же, в свою очередь, дуется на кефирщиков, за то, что они устроили террор архивом с маленьким пряником. Осталось выяснить, у кого больше причин дуться - у оп-а, который всегда лояльно относился к кефирщикам или у кефирщиков, которые вообще непонятно какого хуя устроили тот террор.
джва кефира.jpg91 Кб, 1000x710
100 729023
Кефир это вкусно!
флешка.PNG58 Кб, 1149x1010
101 729024
>>29007

>оп-ходячаяфлешка


Загуглил. Какие-то у вас тут местные разборки. Опять прянефила с кем-то путают. Еще раз, прянефил сидит в этом треде с августа этого года. Меньше, чем полтора месяца всего. Вон даже с ОП-пикчей в прошлом треде >>724642 (OP) проебался - не знал, что с ней уже перекатывали: >>704732 (OP)
102 729051
>>28648

> откуда эта дурь взялась, что у элементарных частиц нет внутренней структуры?


На доступных масштабах энергий (мощности ускорителей) ее не наблюдается.

>Раньше ж так же говорили об атомах, а там потом оказалось вон сколько внутри всего интересного.


На доступных тогда масштабах энергий ее не наблюдалось.
103 729052
>>28716

> Очень просто. Элементарные частицы описываются простым набором квантовых чисел: масса, заряд, спин, время жизни и т.п. Никакая внутренняя структура здесь не требуется.


Кирпич описывается простым набором чисел: длина, ширина, высота, масса. Никакая внутренняя структура здесь не требуется.
104 729056
>>29052
Довнич, спок.
105 729057
>>29056

> Довнич



Серб? Хорват?
106 729062
>>29057
Лол, кстати правда что-то южнославянское слышится.
107 729096
>>29014

>у кого больше причин дуться - у оп-а, который всегда лояльно относился к кефирщикам


Ты кем блядь себя возомнил?
Засунь себе в жопу свою лояльность )
Катись ты сука лесом и варись в своём негромексиканском говне.

Какого хуя модераторку на спейсаче выдали психически больному программисту - который тащит во все треды прянефагготрию и которму насрать на космос?
108 729097
>>29096
Оп-ом треда. Остальное всё - твои домыслы.
109 729103
>>28716
Почему у одних частиц заряд положительный, а у других отрицательный? Из-за внутренней структуры, например. Не?
110 729105
>>28851
Свет и движется по искривленным массой тела траектории. В этом суть эффекта. Если бы не было искривления пространства, то и эффекта гравитационного линзирования не было бы.
>>28853

>И пытаться эту математику натягивать на физический мир


Эта математика как бы для описания физического мира и придумана.
>>29052
Длина, ширина и высота складываются из атомно-молекулярной структуры вещества. Как раз без внутренней структуры объект не может обладать этими характеристиками. Так что ты со своей тухлой аналогией, очевидно, насрал себе в штаны. Еще и потому, что аналогия - не аргумент.
111 729106
>>29103

>Из-за внутренней структуры, например. Не?


Никакой внутренней структуры и элементарных частиц не наблюдается в эксперименте, при этом зарядом они обладают (если речь идет о заряженных частицах). Значит, заряд частицы никак не связан с внутренней структурой. Это просто сечение взаимодействия частицы с электромагнитным полем. У электрона оно такое, что он несет отрицательный заряд. А нейтрино, например, вообще электромагнитно не взаимодействует, поэтому заряд у него нулевой.
112 729107
>>29106
*у элементарных частиц
быстрофикс
113 729115
>>29106
Да нет же. Ну то есть бля. Чому электрон не может существовать без заряда? Просто томущо? Нет. Из-за внутренней структуры.
114 729118
>>29115

>Чому электрон не может существовать без заряда?


Потому что он электромагнитно взаимодействует с определенным сечением.

>Из-за внутренней структуры.


У него нет врутренней структуры.
115 729119
>>29118

>Потому что он электромагнитно взаимодействует с определенным сечением.


Почему он электромагнитно взаимодействует? Из-за внутренней структуры же!

>У него нет врутренней структуры.


У него есть врутренняя структура, например.
116 729120
>>28648
Да хуйня это все. Проблема в том что обезьяний мозг ломается когда доходит до размышлений о бесконечности вещества в обе стороны от точки отсчета. Как в сторону макро так и в сторону микромира. При чем это он для наблюдателя микромир, но так же наблюдатель может быть смещен в сторону того что мы называем атомами и элементарными частицами и для такого наблюдателя все эти сущности будут вполне себе макро. Кирпич можно пилить на две половинки бесконечно. Это ломает мозг.
117 729131
>>29119

>У него есть врутренняя структура, например.


Пруф?
118 729157
>>29131
Да.
119 729172
>>29119

>Почему он электромагнитно взаимодействует? Из-за внутренней структуры же!


Каким образом ты связал одно с другим? Примерно почувствовал?
А какая внутренняя структура у фотона?

>У него есть врутренняя структура, например.


И тут ты такой с результатами экспериментов, показывающих наличие у электрона внутренней структуры, да? Или так, пиздишь заведомую хуйню, лишь бы спорить?
>>29157
Хуй на.
120 729184
>>29172
Да нет же. Ты все в кучу смешиваешь. Фотон - это маленькая порция энергии. У энергии нет структуры.
p6IokUkQthl90uus.mp44,8 Мб, mp4,
1280x720, 1:13
121 729185
Нашел только что картинку со спинами плонет. Объясните, откуда их 10 штук? Вроде ж Плутон и Нептун выписали их плонет еще 10 лет назад, не?
122 729189
>>29185
Нептун никогда не выписывали, а Цереру никогда и не записывали.
123 729198
>>28640 (OP)
Какие книжки почитать про космос? Можно энциклопедию, но какого автора? В детстве читал обобщенную общую энциклопедию с темой космического пространства, строении солнечной системы и проч.
20220919010317.jpg634 Кб, 1080x2220
124 729215
>>29184

>Фотон - это маленькая порция энергии.


Любая частица несет какую-то энергию. Фотон - это элементарная частица, переносчик электромагнитного взаимодействия. Так какая же у него структура?
Ну и твое молчание по поводу экспериментальных данных о внутренней структуре электрона, я так понимаю, является положительным ответом на вопрос о том, пиздишь ли ты тут заведомую хуйню, лишь бы просто поспорить. Впрочем, я не удивлен. Долбоебы типа тебя здесь довольно частое явление.
125 729217
>>29215
Там изначально вопрос был в том, что откуда уверенность что у элементарных частиц нет внутренней структуры, если ее на данный момент не обнаружили. Тебе красненьким даже отметил на скрине: >>28658 . Нахуя ты своим "рряяяя ниси икспиримент" влез?
44089.jpg125 Кб, 1213x795
126 729222
Зачем существует этот троп про выкидывание трупов с корабля в космосе? Разве не разумней трупы немного заморозить за бортом, а потом вернуть на Землю? Вместо того чтобы выкидывать как мусор за борт бессовестно? Вдруг на Земле есть кто-то кто хотел бы похоронить человека, а вы такие " А мы его нахуй из аэрлока выкинули, его труп там где-то летает сейчас"
127 729223
>>29222
Ну это тип как моряков принято было хоронить.
128 729224
>>29222
В космоперах все тырили с морского флота (даже линейный бой) и так прочно закрепилось в произведениях "посерьезнее".
Безымянный.jpg40 Кб, 800x600
129 729230
Не понял, а как сделать камеру Обскура? Был сейчас в темном помещении с некоторым количеством мелких дырок в стене, с другой стороны не было источников света, кроме окна и через дырки проходил только отраженный свет, но вместо перевернутого изображения были тупо слабые, мерцающие в пыли, линии света, с тусклой точкой на конце.
130 729231
>>29230

>с некоторым количеством мелких дырок


Уже в этом проблема.

>с другой стороны не было источников света, кроме окна


И конечно изображение будет ну очень тусклым, поэтому никаких кроме быть не должно.

>как сделать камеру Обскура?


Проще и удобнее всего открутить от чего-нибудь (фотоаппарата, веб-камеры) объектив и вместо него поставить крышку с маленьким отверстием. У фотоаппарата выдержку побольше придется поставить. А вебку придется направлять только на яркие и контрастные объекты, например на соседний дом.
131 729232
>>29222
В реальности их бы не выкидывали, а ПЕРЕРАБАТЫВАЛИ.
132 729233
>>29217

>Там изначально вопрос был в том, что откуда уверенность что у элементарных частиц нет внутренней структуры, если ее на данный момент не обнаружили.


Отсюда и уверенность, чмо. Тот же электрон известен еще с XIX в., и совершенно никаких намеков на наличие у него внутренней структуры не обнаружено до сих пор.

>Нахуя ты своим "рряяяя ниси икспиримент" влез?


Хуесоска тупорылая, это же ты влезла в чужой разговор. Результаты эксперимента я просил у долбоеба выше, который прямо утверждал, что у электрона непременно есть внутренняя структура. Поэтому ты идешь на хуй, как и он.
133 729289
>>29283 (Del)

>Например сам взрыв сверхновой длится дни, часы, или годы?


Вроде пару недель он может длится.
134 729291
>>29283 (Del)
Задумались.
Сверхновая с коллапсом в релятивистский объект длиться всего лишь несколько минут, по мере отражения ударной волны. Остальное это разлетание оболочки с последшим падением яркости, которое будет длиться тысячи лет.
Сверхновые с термоядерным взрывом белых карликов вообще длиться секунды, а иногда меньше. И тут так же как в предыдущем случае формуется разлетающийся облако.
Образование ЧД не ясно в деталях, ибо не проверенной теории квантовой гравитации, но если брать по классике, то время формирование ЧД это момент формирование ложного горизонта и становление его истинным.
Ложный горизонт формируется заранее и делает со скоростью света, скрывая от нас процесс формировании сингулярности, который уже содержит много произвольного. Так сингулярность может возникнуть сразу, а может для этого потребуется бесконечное количества времени и истинный горизонт никогда не сформулируется. Тут уже нужны детали, который мы скорее всего никогда не узнаем.
Сжатие звезд в белого карлика идет в ее недрах на протяжение всей жизни звезды. Правда есть стадия, когда гидростатическое равновесие будет нарушение и термоядерные реакции будет идти на сброс оболочки - это уже стадия красного гиганту, этот будет продолжаться сотни тысяча лет в зависимости от звезды.
135 729325
>>29291
Не понял насчет ложного горизонта и при чем тут квантовая гравитация?
136 729344
>>29325
Ложный горизонт событий называют ложным, потому что из под него в будущем таки можно вывести из под него геодезическую, хотя локально(по времени) он не отличим от истинного горизонта. Короче упав под него таки есть способ вернутся назад. Горизонт событий остается ложный, пока не создаться сингулярность под ним, породив истинный горизонт, который плавно сольется с ложным.
Процесс зарождения сингулярности уже не описывается ОТО, тут просто необходимо учитывать квантовые эффекты. Поэтому чтоб узнать когда ложный горизонт станет истинным нам нужна квантовая гравитация.
137 729383
>>29283 (Del)

>взрыв сверхновой длится дни, часы, или годы? Или образование черной дыры? Сжатие звезды до карлика?


Все эти процессы по меркам человека происходят моментально, за секунды или доли секунды.
138 729413
>>29344
Но гравитация - это искривление пространства-времени. При чем тут вообще кванты? И как они вообще могут помочь?
139 729416
>>29413
Тут есть несколько причин.
-Первая. Наша Вселенная квантовая, а это значит любой физический процесс порционнен или его физическое действие дискретно, оно не может быть сколько угодном малым. ОТО же просто геометризация всех физических процессов и это делает это через действие. Раз действие дискретно и ОТО просто обязано квантоваться. В ЧД под горизонтом событий физическое действие любой системы становится слишком чувствительным к геометрии и наоборот, ну и соответственно там начинаются цвести всякие квантовые эффекты буйным цветом.
-Второе. ОТО требует строгой непрерывности пространства-времени, в ней все должно быть гладко и непрерывно. Если и есть сингулярности, то они жестко закреплены топологией или являются устранимыми. Коллапс ЧД же порождает сингулярность скачком. Ни как не получается ее плавно сделать. Однако у нас Вселенная квантовая и геометрия на малых масштабах меняется скачками, то и рождение сингулярности автоматом снимается.
140 729417
>>29416

>дискретно


Ну это еще не доказано, на самом деле.
141 729419
>>29417
В смысле не доказано?
На дискретности действия строится вся квантовая теория, это ее главный столб. Атом как таковой является главным доказательством этого.
142 729425
>>29419

> В смысле не доказано?


в прямом, нет ни доказательств, ни двже обоснованных предпосылок, ни один эксперимент не показал прямо или косвенно что пространство-время дискретно
143 729434
>>29425
Я говорил про дискретность действия, а не пространства-времени. Даже если переводить его дискретность в геометрию, это не будет означать дискретность пространства-времени. В квантовой теории к действию добавляется безразмерный множитель - фаза, который обеспечивает непрерывность для всех нужных величин. Так электромагнитные поля непрерывны, но меняются дискретно через обмен фотонами. С ОТОшной гравитацией так же, гравитационные поля(искривление пространства-времени) непрерывно, но его изменение идет через поглощение или испускание гравитационных волн, что можно переформировать как обмен гравитонами. Гравитационное излучение уже зарегистрировано, а это уже большое подспорье о существование гравитонов.
144 729436
>>29434

>испускание гравитационных волн, что можно переформировать как обмен гравитонами


Не обязательно.
145 729437
>>29434

>Гравитационное излучение уже зарегистрировано, а это уже большое подспорье о существование гравитонов.


Когда их найдут в эксперименте - на БАК или еще где-либо, тогда и можно будет говорить об этих твоих гравитонах как о чем-то реальном. А пока это лишь пердеж в лужу.
ATSxf4t5lsl0P1C.mp45,2 Мб, mp4,
1280x720, 1:13
146 729549
>>29185
Кек блять)
147 729582
>>29434

>Гравитационное излучение уже зарегистрировано


Лол, нет. Как не зарегистрированы и гравитоны. А потому что нет никакого "гравитационного излучения"
148 729583
>>29434

>Гравитационное излучение уже зарегистрировано


Ты наверно хотел написать "гравитационные волны зарегистрированы"?
149 729584
>>28971
давай рассуждать логически, понимаю, трудно, но попробуй.

радиотелескоп фиксирует излучение исходящее от звёзд?

да

когда планета проходит между звездой и наблюдателем (нашим телескопом) то она поглощает часть излучения звезды?

поглощает

отсюда вывод
150 729682
>>29584
Как ты отличишь темное пятно на поверхности звезды от планеты?
151 729741
>>29682
ну как бы пятна они пропадают и не стабильны, а планета обращается с одинаковой периодичностью
152 729762
>>29682
Какое ещё темное пятно?

Смотрят на интенсивность излучения, если она с регулярной периодичностью кратковременно падает - можно сделать вывод, что это планета проходит между звездой и телескопом.
дедов микроскоп.jpg100 Кб, 900x874
Клетки Фарадея тред #1 153 729789
Ребзя, вы ж тут дохуя шарите, не? Ну там физики, шизики, типа. Объясните, чому "клетка Фарадея" не взлетает? Есть короч чугуниевый футляр от дедовского микроскопа, как на картинке, только более ржавый (магнитик липнет). Микроскоп 1948-го года, серийный номер - N37809 (чому он у них на латинском?). От чугуниевой подставки (магнитик липнет) открутил микроскоп и коробочку для дополнительных линз. Отверстия в подставке заделал фольгой. Ну и еще сам футляр фольгой чутка обклекил, хз зачем. Короч, два вопроса:
1. Надо ли его заземлять об батарею?
2. Положил туда внутрь телефон (Galaxy A7 2018). Между телефоном и подставкой - 3 см диэлектрика. До стенок футляра от телефона - сантиметров 5. В закрытом футляре телефон не пропадает из сети и получается на него позвонить. Чому эта хуйня не работает?
20220927193805.jpg3,2 Мб, 5664x4248
154 729790
>>29789

>Микроскоп 1948-го года, серийный номер - N37809

155 729822
Ну бля. Хули вы как это? Срочно надо же)
156 729823
>>29789
У чугуния хуевая электропроводность.
Для клетки фарадея нужен материал по электропроводности хотя бы на уровне алюминия.
157 729828
>>29823
То есть от EMP эта хуйня не спасет? Взять просто обувную коробку фольгой обклеить?
158 729833
>>29828
Нет, не спасает.
Фольгу нужно достаточно толстую.
202209280014211.JPG315 Кб, 1725x1402
159 729837
>>29833
Коробочка от печеньки сгодится? Не алюминиевая - магнитится. Ну и опять же, надо ли заземлять?
160 729838
>>29789

>Чому эта хуйня не работает?


Потому что размер ячейки в сетке клетки должен быть намного меньше длины волны. А у тебя по периметру контакта нет, вот твоя ячейка сетки, которая получилась больше длины волны.

>Надо ли его заземлять об батарею?


Поскольку соединение с землей явно больше длины волны, то его нужно считать как длинную линию. А поскольку импедансы у этой длинной линии несогласованны, то импеданс такой линии будет сильно отличен от нуля, да еще будет от частоты меняться. Поэтому на таких частотах таким образом соединять с землей бессмысленно.

Анона выше не слушай.
00014.mp415,2 Мб, mp4,
1440x1080, 1:22
161 729860
>>29838
Ну ок, скажем диапазоны частот мобильной связи - 800 МГц, 900 МГц, 1800 МГц, 2100 МГц и 2600 МГц. Чтобы узнать длину волны для частоты, надо прост взять расстояние (в метрах), которое проходит свет за секунду и разделить его (расстояние) на эти самые герцы. Узнаем, сколько раз колебался вектор электромагнитной индукции на отрезке в один метр. Расстояние между двумя амплитудами колебаний будет той самой длиной волны, не? Итого, длины волн для указанных частот - 0.375, 0.333, 0.166, 0.142, 0.115 в метрах.
То есть, даже для самой охуевшей связи в 2.6 ГГц, достаточно клетки с размером ячейки меньше 11.5 см, чтобы экранировать сигнал? 11.5 см - че-т дохуя, нет? В такую ячейку голова пролезет.

>намного меньше


"намного" - это как математически выразить?

Коробка от печеньки тож нихуя не работает (пикрелейтед). Кстати, еще вопрос. Чому коробка от печеньки тож нихуя не работает?
162 729861
>>29838

>Поэтому на таких частотах таким образом соединять с землей бессмысленно.


С землей соединять для того, чтобы в батарею все статическое электричество утекло, не? Во время emp весь металл же статикой заряжается, не?
163 729904
>>29860
так вот как выглядит аватаркоблядь срущая филиппинкой в спесикс тредах
малолетка с обгрызенными ногтями
164 729915
>>29904
Чиво блять?
165 730138
>>29185
Напомните почему уран прилёг? В него что-то большое и толстое врезалось?
166 730171
>>30138
Наиболее вероятно версия такая.
Было несколько крупных планетозималей.
Они сычивали где-то на уровне орбиты нынешного Марса.
Но произошел "прыжок Юпитера" и эти планетозимали были выбранены нахуй далеко, где она начала своим еблом собирать газ и льды. Потом произошла их рокировка с Сатурном, и планетозимали начали слипаться в один кусок, попутно выкидывая лишнего вещество, формируя говно, которое будет ебашить Землю потом.
Короче Уран был кратной системой как Земля - Луна, но постепенно слился в один.
Сильный квадрупольный момент магнитного поля косвенно говорит о том, что в нем силикатные ядра так и не слились в одно целое.
167 730219
>>30171
Нда, чёт на русском ютубе нихрена про это нет.
Везде про кольца.
168 730249
>>30219
Гугли планетообзование на ангельском. В любой лекции по экзопланетам так или иначе затрагивается тема образования нашей солнечной системы.
У Попова были лекции по моделям формирования.
169 730322
Какие параметры быстрый так чтобы симуляция сохраняла свою точность
2. чтобы планета вращаясь вокруг солнца того или иного радиуса была в обитаемой зоне, и чтобы при этом они действительно убедиться чтобы на ней действительно могли располагаться океаны и признаки жизни
3. Юпитер будучи солнцем и какой размер должен быть у его спутников и на каком расстоянии так чтобы на них сохранялось возможность жизни как удостовериться.
170 730340
АНОНЫ, А ВЫ ЗНАЛИ, ЧТО НЕЛЬЗЯ ДОЛЕТЕТЬ ДО СОЛНЦА???

Этот вопрос покажется легким для тех, кто играл в Kerbal Space Probram. Но все таки.

Почему нельзя запустить ракету прямо на солнце, долететь до него и упасть (приземлиться) на него?
Почему так? Кто знает?

Сразу же дам ответ, что до солнца все таки можно долететь, но нужно лететь не к нему. Нужно лететь к ОТ НЕГО (к Плутону). Примерно на него орбите затормозить (снизить орбитальную скорость до 0) и ждать тысячи или десятки тысячи лет когда солнце само притянет ваш космический аппарат.

Так и почему нельзя долететь до солнца и упасть на него?
171 730361
>>30340
Иди нахуй со своей жирнотой дегенерат, тебя уже в б обоссали.
16648093577570.png1,1 Мб, 1280x720
172 730376
173 730377
>>30340

>Почему нельзя запустить ракету прямо на солнце, долететь до него и упасть (приземлиться) на него?


Можно. Солярки побольше залей и лети.
174 730384
>>30340

>Почему нельзя запустить ракету прямо на солнце, долететь до него и упасть


Можно.
photo2022-10-0412-55-01.jpg126 Кб, 1280x720
175 730386
Каким-то шизикам дали нобелевку по физике:

>Нобелевскую премию по физике получили Ален Аспект, Джон Ф. Клаузер и Антон Цайлингер за эксперименты с запутанными фотонами, установление нарушения неравенств Белла и новаторство в квантовой информатике.

176 730387
>>30386
Какие шикарны усы у первого дядечки, ему бы телепроповедником работать или продавцом подержанных машин.
photo2022-10-0413-25-30.jpg92 Кб, 680x680
177 730389
image.png448 Кб, 1000x1008
178 730448
>>30377
>>30384
Нельзя вообще-то, Джейсон запретил.
Yemi Lab.jpg83 Кб, 920x627
179 730452
Поясните за Yemi Lab? Пишут, что южнокорейцы построили какую-то ёба-лабораторию на глубине 1к км, чтобы изучать тайны Вселенной.
Гуглится какая-то хуита.
16645693992621.png1,2 Мб, 1280x897
180 730453
Если предположить, что в космосе, на орбите близ марса-земли, возникнет шар массой одну земную из сыра
То какая его дальейшая эволюция? Понятное дело что давление резко начнет расти в недрах а с ней температура, от сил сжатия гравитацией
На что начнет распадаться ядро? какие могут быть вариации состава мантии? Все превратится в нефть? И может на поверхности возникнет глобальный океан из воды или сыра? Какая может быть атмосфера?
Что с составом всего будет через миллион или миллиард лет?
181 730454
>>30453
Яро скорее всего превратится в алмаз, прочее скорее всего в уголь, океаны из смеси воды и углеводородов, атмосфера те же углеводороды, углекислый газ и азот.
182 730455
>>30452
Нейтрино наверно ловят.
183 730464
>>30454
То есть большая часть газов улетит? планета станет на треть меньше?
184 730467
>>30464
Это будет зависеть от разных факторов. Титан вот например имеет плотную атмосферу.
Безымянный.jpg15 Кб, 800x600
185 730526
Хочу написать крипипасту про паралельный мир. Может ли, в теории, быть такая планета, что вся вода притягивается приливными силами Луны таким образом что скапливается только на одной половине планеты? А на другой пустыня с редкими озерами?
186 730530
>>30526
В теории, если планета не вращается и всегда повернута одной стороной к звезде, наверное.
Но чот я не уверен, что такое вообще может быть.
187 730541
>>30526
Гравитация способная стянуть воду на одну сторону разорвет обе планеты нафиг. И в чем криповость океана с одной стороны и пустыни с другой? Того же эффекта можно добиться рельефом планеты.
Tidal-forces-calculated.png5 Кб, 361x403
188 730550
>>30526
Нет, приливные силы создают прилив с двух сторон (поэтому он два раза в день).

>>30530
Чтобы была повёрнута одной стороной к звезде конечно может.
189 730556
>>30530
>>30550
Ок.
>>30541
Да там крипота не в этом. Просто так блокируется путь назад для гг.
190 730567
>>30453
Почитай про планету из кротов от what-if, чтобы примерно почувствовать.
16651601726010.webm2,6 Мб, webm,
404x720, 0:53
191 730644
Увидел в бэ видеорелейтед. Почему в данном случае легкие элементы скапливаются внутри, тяжелые снаружи, а у протопланет было наоборот?
Дело только в меньшей скорости вращения из-за которой гравитация победила?
192 730653
>>30644
Потому что в планетных масштабах есть еще одна хуйня - самогравитация. Тяжелые элементы банально тонут. А в данном случае самогравитацией можно пренебречь, т.к. сила гравитации данного шара на десяток порядков меньше других эффектов (центробежной, молекулярных связей, и т.п.)
193 730675
194 730684
>>30675
Вполне, но если ты хочешь серьезных моченых щщей, то покажется не очень.
195 730697
Если Вселенная бесконечна, то как она может расширяться (как может расширяться нечто бесконечное)? А если конечна, то что-то же должно быть за её пределами? Или там просто ничего нет? Вот конец Вселенной и дальше ничего?
196 730698
>>30697
Как поверхность сферы. Если ты двигаешься по своему глобусу в одну сторону очень долго - через какое-то время вернешься в исходную точку, так и не добравшись до границ глобуса. При этом сфера может расширяться - во всех точках растягивается поверхность.
Вот так же и Вселенная, только там не сфера, а ЧЕТЫРЕХМЕРНЫЙ ТОР (бублик).
197 730699
>>30698
То есть двигаясь во Вселенной с бесконечной скоростью в одном направлении можно вернуться в исходную точку?
198 730702
>>30699
Да.
199 730703
>>30697

>Если Вселенная бесконечна, то как она может расширяться (как может расширяться нечто бесконечное)?


Ты воспринимаешь расширение Вселенной как обычное расширение среды, с которым мы имеем дело в повседневной жизни. Например, расширение газа или поднятие теста. Но обыденная логика здесь не работает. Расширение Вселенной - явление другого порядка, не имеющее аналогий из повседневной жизни. Тут работает метрика ОТО. Если совсем упрощать, то у тебя за каждый промежуток времени (скажем, за 10 лет; число взято с потолка, просто для иллюстрации) расстояние между двумя точками удваивается. Было 10 м, прошло 10 лет - стало 20 м. Т.е. как бы появляется больше пространства из ничего. Это и называют расширением Вселенной. А какая Вселенная топологически - бесконечное трехмерное евклидово пространство или трехмерная гиперсфера - совершенно в данном контексте неважно. Расширение Вселенной никак не зависит от ее топологии, т.к. им управляют иные силы. Иными словами, Вселенной не нужно никакого внешнего пространства, чтобы расширяться.
>>30698

>так же и Вселенная, только там не сфера, а ЧЕТЫРЕХМЕРНЫЙ ТОР (бублик)


И тут ты такой с пруфами, да? Или пруфов у тебя нет, а ты просто примерно почувствовал глобальную топологию пространства?
16638608517610.jpg409 Кб, 1152x1208
200 730714
может ли быть так, что межгалактические сверхпустоты это в бывшем сильно разогретые участки на сверхранних этапах развития вселенной?
Вон по исследованию реликтового излучения заметили флуктуации его интенсивности, быть может эти участки (вещество в них) в процессе инфляции вселенной стремились разлететься быстрее чем другие, а галактические нити просто места столкновения вещества из этих разлётов
201 730720
>>30714
Тащемта наоборот, войды это самые среднее участки, а локальные экстремумы породили зародыши роста для концентрации ТМ, а нити уже сформировались как соединение этих концентраций.
202 730723
>>30714

>а галактические нити просто места столкновения вещества из этих разлётов


Нити сформировались в результате флуктуаций плотности вещества в ранней Вселенной. Сначала стало комковаться темное вещество, потом туда натекло обычное вещество. Так возникла крупномасштабная структура, частью которой являются нити.
203 730769
НАСКОЛЬКО ЯРКО СВЕТИТ РУНА
Почему так ярко? Там же гранит/грунт/пыль, а не белый лед????

щас полная луна
светит даже не желтым а белёсым-бело-голубым, и светит так сильно что отбрасывает тень от предметов будто футбольног стадиона прожектор с расстояния около километра. НО ДО ЛУНЫ 300 000 км!!!!!

какая же там светимост? и как Аполлоны не слепли????
204 730770
>>30769

>РУНА


ЛУНА
205 730787
>>30769
На самом деле Земля "светит" т.е. отражает лучи Солнца сильнее чем Луна. Т.е. если ты окажешься на Луне, то Земля будет такая яркая что ты даже мальца кекнешь.
206 730788
Существует ли галактика в форме пениса?
207 730789
>>30702
Что "да", говна кусок?
208 730790
>>30788
ясен хуй что существует и не одна и в форме пизды тоже
ты же представляешь сколько милилардов миллиардов галактик во вселенной? там любые формы попадаются
столкнулись две галактики образовались яйца, к ним подлетела третья, её гравитация двух других вытянула и получилось хуй с яйцами, даже где-нибудь скопление таких галактик есть
209 730791
>>30769

>ЯРКО СВЕТИТ РУНА


луна не светит, от неё отражается солнечный свет, вот оно ярко светит пиздец как, если ты не заметил вдруг
210 730792
>>30769

>какая же там светимост?


хуёвая

>>30769

>и как Аполлоны не слепли????


ты по ходу думаешь, что с приближением к луне, яркость её увеличивается
но тогда ты действительно тупой
мега блять тупой
211 730795
>>30792
хоть один довод почему с прилижением к луне ее ярксоть не будет увеличиваться?
закон кубов-квадратов так-то
212 730803
>>30769

>Почему так ярко?


Так лишь кажется, потому что на фоне черное ночное небо. У Луны очень низкое альбедо, она поглощает большую часть солнечного света, поэтому она очень тусклое тело. Поверхность Луны очень темная.
213 730812
>>30769
Она ярко светит, потому что у глаза адаптивная чувствительность, твой зрачок сужается и расширяется в зависимости от освещённости. Поэтому когда нет других источников света, кажется, что Луна светит ярко. На самом деле у неё довольно низкое альбедо.
584392mainM168000580LRap17area.jpg997 Кб, 1600x1200
214 730814
>>30795

>почему с прилижением к луне ее ярксоть не будет увеличиваться?


Пикрил.
И прикинь какая земля яркая с луны. Как только людишки не слепнут на ней?

>>30795

>закон кубов-квадратов так-то


Что за хуйню ты несёшь? Ты понимаешь, что ты поехавший?
215 730818
>>30814

>Что за хуйню ты несёшь? Ты понимаешь, что ты поехавший?


В любой непонятной ситуации говори ЗАКОН КУБОВ КВАДРАТОВ и делай умное выражение лица.
216 730825
>>30795

>закон кубов-квадратов так-то


Я знаю только закон ниггерских хуев в пизде твоей мамаши.
217 730851
>>30795
Подсказка, сама Луна только отражает свет.
Подсказка два, вспомни Луну на дневном небе.
218 730982
Вот мы находим все более древние галактики, а что с противоположной стороны? Такие же старые галактики? Но мы же не находимся в центре вселенной. Т.е. условный "край" вселенной с какой-либо стороны для нас находится ближе или как? Пытался нагуглить, но уверен, что неправильно ставил вопрос.
И посоветуйте, что можно почитать на подобную тему
219 730995
>>30982

>Вот мы находим все более древние галактики, а что с противоположной стороны? Такие же старые галактики?


С "противоположной стороны" находится причинно не связанная с нашей наблюдаемой областью часть Вселенной. Там такие же галактики, как и здесь, среди которых есть как очень старые, так и довольно молодые.

>Т.е. условный "край" вселенной с какой-либо стороны для нас находится ближе или как?


У Вселенной нет никакого края - ни условного, ни какого-либо еще. Причина, по которой мы находим все более древние галактики, совершенно прозаична и заключается только в том, что скорость света конечна. Поэтому чем дальше мы смотрим в пространство, тем более старые галактики мы видим. Свету, испущенному этими галактиками в разные моменты их жизни, требуется большое время, чтобы достичь наблюдателя, особенно с учетом расширения Вселенной. Это никак не связано ни с какими краями или с тем, что мы находимся в каком-то выделенном месте.
220 731006
>>30995
Недавно начал интересоваться, не обессудь. Это я понимаю, читал про свет и красное смещение. Хотя стало интересно, как учитывают расширение вселенной в измерениях возраста какого-либо объекта. Вопрос в другом был, либо я тупой просто и себя путаю. Почему вселенная выглядит одинаково во всех направлениях.
Немного ломается мозг от того, что нет "центра" у вселенной, либо мы этого не знаем. Хотелось прочитать более простой ответ для "дурачков".
Но спасибо за ответ)
221 731011
>>31006
Простых ответов для дураков не будет.
Даже термин расширение вселенной является изменение масштабного фактора, а не расширение газа и чего еще.
Если хочешь вкатится тебе надо понять, что такое пространство-время и какие операции в нем, а потом уже ОТО и модели Вселенной.
Если можешь в матан то накатывай трехтоник Мизнера/Торна "Гравитация" и Вейнберга "Гравитация и Космология".
222 731013
>>31011
Благодарю, буду читать
223 731035
>>31006

>Почему вселенная выглядит одинаково во всех направлениях.


А почему она должна выглядеть по разному? Вселенная более или мене равномерно заполнена веществом и расширяется с равномерным ускорением

>Немного ломается мозг от того, что нет "центра" у вселенной, либо мы этого не знаем. Хотелось прочитать более простой ответ для "дурачков".


Попробуй найти центр поверхности Земли.
anime5.png5,1 Мб, 2893x2893
224 731036
>>31035
Мне нравится аналогия с воздушным шариком. Когда он надувается, в каждой точке его поверхность растягивается одинаково, и нет никакого "центра" растягивания. Вселенная расширяется аналогично, только в богомерзком 3D вместо няшного 2D.
225 731047
>>31006

>Почему вселенная выглядит одинаково во всех направлениях


Потому что она однородная и изотропная. В ней нет выделенных мест и направлений.

>Немного ломается мозг от того, что нет "центра" у вселенной, либо мы этого не знаем


Мы знаем точно, что его нет.

>Хотелось прочитать более простой ответ для "дурачков"


Ну представь сферу - поверхность шарика. Представь, что на сферу нанесены точки - галактики. Теперь представь, что эта сфера начинает растягиваться одинаково во всех своих точках. Ты будешь наблюдать точно такой же эффект, как и в реальной расширяющейся Вселенной - все "галактики"-точки будут удаляться от тебя. При этом если ты перепрыгнешь из одной такой галактики в другую, то будешь наблюдать ровно ту же картину. Как раз потому, что у сферы нет центра, нет выделенных направлений - она растягивается вся и сразу. То же самое и в нашей Вселенной, с той лишь разницей, что мы живем не в двумерной сфере, а в трехмерной гиперсфере.
226 731053
>>31047
Аналогий с шариком лютая хуита по многих пунктам.
1. Совершенно разные топологии поверхностности сферы и космологического пространства-времени.
2. Расширение это измерение масштабного фактора по направлению, а не локальная геометрия. Короче это как растяжение(на самом деле корректно назначать сжатием или скукоживанием) координатный сетки, а не самого многообразия. такие финты никак не показать на шарике.
3. На 2-мерном вещественном многообразии нет рабочих когомологий. Эж ебал сферу.
227 731065
>>31053

> Расширение это измерение масштабного фактора по направлению, а не локальная геометрия


а нити и войды не про локальную геометрию при расширении?
228 731079
>>31053
Для создания у далекого от темы человека представления на самом базовом уровне эта аналогия вполне годится. Недаром эту аналогию используют всегда, когда хотят объяснить неподготовленной аудитории явление расширения Вселенной. К ней прибегал даже Капица на своих университетских лекциях.
depositphotos165669000-stock-photo-two-glasses-of-tea-with (1).jpg340 Кб, 1600x1700
229 731092
230 731144
Радиус черной дыры - что такое?!

У нее ведь нет размера, это просто точка синуглярности?
16638673266410.png386 Кб, 512x512
В чем неправ этот писатель? 231 731146
... 2065 год - изобретение компактного реактора, работающего на гелии-3. Начало новой космической эры (НКЭ).

2095 год - основана первая колония на Луне, численностью 50 жителей.

2105 год - основана первая колония на Марсе, в ледяном кратере [данные удалены].

2155 год - лунный радиотеоескоп "Хаумеа" засекает мощный радиосигнал. Радиосигнал являет собой: 1% - простые числа от 1 до миллиарда; 99% - сложная цифровая комбинация.
Попытка расшифровать сигнал провалилась.

2250 год - в связи с подтверждением теории о существовании внеземной жизни государва ООН и ООП (организация объединённых планет) решают создать "Солазер" - рой отражателей на близкой орбите Солнца, способных сконцентрироваться излучение и уничтожить пришельцев в случае их вторжения.

2290 год - "Солазер" завершен. Создано два контрольных пункта оружия:
1) Земля, Аляска, территория бывшых США.
2) Система Юпитера, спутник Ганимед,
Точное местоположение обоих пунктов засекречено. Существование второго пункта совершенно секретно.

2300 год - вооружённое нападение террористической организации "Независимая Луна" на базу контроля Солазера на Земле.
После продолжительного боя, террористы захватывают контрольный пункт и передают его координаты Солазеру.
Удар Солазера, достигший Земли через 60 минут, уничтожает все живое в радиусе 1000 км от места удара и вызывает дестабилизацию климата, а также геологичесй активности планеты.

2330 год - после "Гибели Земли" государства Марс, Луна, Меркурий ведут холодную войну и исследуют Солнечную систему.
Во время одной из исследовательских миссий зонд "Ганимед-орбитер" обнаруживает чертания рукотворного объекта. Вероятно - того самого контрольного пункта "Солазера".

Государства Меркурий, Луна и Марс решаются бросить в бой все силы, чтоб захватить контрольный пункт и обрести власть над Солнечной системой.

Одновременно с этим происходит окончательная расшифровка инопланетного сигнала группой марсианских учёных...
232 731147
Анончики, мне очень нужна ваша помощь! Срочно! Про челябинский метеорит.
Есть очень серьезные расхождения в информации и наблюдаемых явлениях.
Согласно официальным данным, размер метеорита составлял не более 18м диаметр и 13000т масса. Однако.
Высота взрыва болида - 25км, а мощность - не более 1.5мт по официальным данным. Каким образом ударная волна от взрыва мощностью 1.5 мегатонны, преодолев более 40 км в разреженной атмосфере (а эпицентр был на высоте более 20км) смогла повредить столько сооружений?
Есть сайт, где даются более приближенные к действительности оценки.
www.synerjetics.ru/article/border.htm
Согласно им, метеорит был диаметром минимум 180м, массой более 1.5млн тонн, а мощность взрыва составила 57мт. Это кажется излишним, но можно проверить и на сторонних ресурсах - эти оценки больше подходят под наблюдения.
Вопрос в следующем.
Если метеорит был аж 180 метров, то почему его не заметили? В тот же день ожидалось сближение с метеоритом меньшего диаметра. Если метеорит был изо льда и снега, и летел со стороны Солнца, то где след от испарения его вещества?
Ну и еще один вопрос, почему официальные данные столь занижены. Едва ли можно поверить в ошибки в расчетах. В чем причина?
233 731149
>>31147

> а мощность взрыва составила 57мт



Это прям "Царь-бомба" какая-то...
234 731153
https://atomlimbs.com/
Что думаете?
235 731160
>>29105

>Еще и потому, что аналогия - не аргумент.


Ну вообще-то аналогия может быть аргументом, но не может быть доказательством.
236 731161
>>31144

>Радиус черной дыры - что такое?!


Расстояние от центра до горизонта.
>>31147

>Едва ли можно поверить в ошибки в расчетах. В чем причина?


Заговор иллюминатов, не иначе.
237 731162
Пасаны, а есть кто с юга? Прямо под луной сейчас красная звездочка, прям оче красная, других такого цвета не вижу, да и довольно яркая.
Что за звезда? Или это Марс?
238 731164
>>31161

>Заговор иллюминатов


Это понятно :)
Меня интересует матчасть. Например, никто не хочет признавать, что по такой траектории летают метеориты диаметром 200 метров. Возможно, есть другие объяснения.
239 731165
>>31162

> Или это Марс?


Он самый
240 731166
>>31165
Его действительно видно невооруженным глазом? Мне казалось что раз он маленький и атмосферы почти нет, то свет плохо отражается и глазом не увидеть.
241 731167
>>31166

>Его действительно видно невооруженным глазом?


Планеты довольно яркие на небе. В идеальную погоду без светового загрязнения можно и уран увидеть.
242 731169
>>31161
А почему горизонт имеет расстояние?
Центр и внутренний край горизонта разве не должны быт в одной точке("сингулярности")?
243 731170
Есть ли предел максимальной массы чёрной дыры, основанный количестве потенциальной энергии в видимой вселенной, необходимой на собирание всей массы в одном месте, и на времени, которое осталось до того, как тёмная энергия начнёт всё расталкивать? ну кроме твоей мамки
244 731171
>>31169
Горизонт это место в пределах которого вторая космическая скорость больше скорости света.
245 731189
>>31147

>смогла повредить столько сооружений?


Единственное поврежденное сооружение это насколько я помню какой-то старый заводской цех. Там побило стекла, причем в основном старых деревянных рамах, стеклопакеты в пластике почти не пострадали.

>Согласно им, метеорит был диаметром минимум 180м, массой более 1.5млн тонн


Это больше аризонского метеорита, там весь Челябинск снесло нахер.
Так что я думаю официальным данным вполне можно доверять.
246 731190
>>31170
По ОТО нет верхнего предела.
Из общих соображений масса(именно масса, а не энергия. это важно, внезапно она зависит от параметра давления и температуры) всей наблюдаемой вселенной
В рамках инфляционной теории, ограничение задается конденсатом инфлатонного поля, но с ним много проблем.
247 731191
>>31169
А почему он не должен иметь?
Центр и сингулярность это разные вещи.
Центр это грубо говоря относительно далекого наблюдателя по пространственной гиперплоскости очерчиваем область горизонтом событий и находим там геометрический центр.
Сингулярность же точка в пространстве-времени, куда сваливаются все мировые линии. Из-за то, что пространство-время кривое, проекция сингулярность на какую-либо ось/гиперповерхность может быть не точкой.
248 731192
>>31191

>Из-за то, что пространство-время кривое, проекция сингулярность на какую-либо ось/гиперповерхность может быть не точкой.


>


как понять
249 731193
>>31190
Да не, я имею в виду, чтоб мы с максимальным кпд и скоростью стягивали вещество со всей видимой вселенной на Землю в одну большую чд. На это и времени дохуя уйдёт (если мы полетим с 99.9% скоростью света на край вселенной, докуда максимум доберёмся, чтоб потом вернуться обратно?), и энергии-массы придётся кучу рассеять вовне, чтоб оставшейся массе дать импульс в нашу сторону, которая от сжатия тоже греться будет и нужно будет заталкивать сильнее наверно?
250 731194
>>31192
У нас в ОТО 4-мерное пространство-время. По ряду причин мы его не можем все в целом рассматривать, вместо этого фиксируемым в нем некоторое направление и делаем этакий "срез" или проекцию. В самом простом случае этот срез представляется привычный нам 3д мир в какой-то моменте.
Поскольку пространство-время кривое, то и срез пидорасится. И когда пидорашение слишком сильное, одна единственная точка может размазаться в произвольную кривую или даже область.
251 731196
>>31194

>кривое


что это значит? а некривое - какое было ы?
252 731197
>>31161

>Расстояние от центра до горизонта.


Нихуя подобного.
>>31144

>У нее ведь нет размера, это просто точка синуглярности?


А хрен его знает. Они точно меньше нейтронных звезд (пара десятков км в диаметре) и вероятно не меньше планковской длинны. Если понятие "размер" вообще применимо к сингулярности.
253 731203
насколько же медленно распротсраняется звук?????

тольок щас посчитал что условно взрыв самолета на расстонии 10км от тебя ты услишиь лишьчерез полминуты....
я про боинг над донбассом
почему так медленно?

а еще я вырос или flightradar врёт: раньше я не видел самолеты на высоте 10км невооруженным глазом, а сейчас это вполне крупные физиески различимые модели. Как будто 200м самолет на высоте 5 км мб, а оказывается это 37-метровый боинг на высоте 10.5 км. А кажется таким большим. А в детсстве/юности я помню самолеты на небе лишь крошечными точкам, даже самолетов не помню. лишь следы их
254 731206
>>31203
1 В воздухе - примерно 300 метров в секунду.
2 Возможно в детстве тебе попадались какие-нибудь мелкие советские самолеты.
255 731207
>>31196
В общем смысле кривое пространство это значит, что если ты пройдешь замкнутый путь и вернешься в изначальную точку, то тебя развернет на некоторый угол. Некривое (плоское) это когда тебя не разворачивает.
В пространстве-времени не возможно легально мировую линию сделать замкнутой, но в дифгеометрии вместо замкнутого пути можно использовать некоторую гиперповерхность, контуром которой является несколько мировых линий. Кривизна соответственно тут уже является не угол поворота, а более сложное действие - деформация площади гиперповерхности. В плоском пространстве-времени деформации не будет.

Ну или совсем по другому. В некоторой области из некоторого события пускаем мировые, которые на границе области "разворачиваются" и соответственно они обратно сходятся в другом событии. В некривом случае мировые линии сойдутся в одном событии, в кривом соответственно мирровые сойдутся в одной точке.
256 731211
>>31203

>а еще я вырос или flightradar врёт: раньше я не видел самолеты на высоте 10км невооруженным глазом, а сейчас это вполне крупные физиески различимые модели. Как будто 200м самолет на высоте 5 км мб, а оказывается это 37-метровый боинг на высоте 10.5 км. А кажется таким большим. А в детсстве/юности я помню самолеты на небе лишь крошечными точкам, даже самолетов не помню. лишь следы их



Еще не забывай что воздушные коридоры могут менятся, в тч по высоте. Например у меня внезапно начали с весны самолеты летать. Ну я конечно подумал что военные, оказалось что нифига, теперь тут большой поток рядом судя по флайтрадару. Более того, этот поток вообще гигантский, но того что летает на 12 км не слышно и почти не видно.
Еще кстати звук тоже от атмосферы очень сильно зависит - иногда звук доходит с дохуя а иногда нет, на больших расстояниях звук идет далеко не по прямой.
257 731214
>>31169

>А почему горизонт имеет расстояние?


Горизонт - это условная граница, которая отделяет внешнюю Вселенную от области пространства, для которой вторая космическая превышает скорость света. Собственно, это и есть ЧД. За горизонтом - пустое пространство, которое упирается в центр - сингулярность. Попав за горизонт, любой объект неотвратимо падает в центр. Ну и, логично, от горизонта до сингулярности лежит определенное расстояние. Это и есть радиус черной дыры. Даже без всяких горизонтов и сингулярностей радиус ЧД - это элементарное понятие. Для внешнего наблюдателя ЧД выглядит как абсолютно черный шар (точнее, сфероид). У этого шара, как и у любого другого, существует определенный радиус. Все просто.

>Центр и внутренний край горизонта разве не должны быт в одной точке("сингулярности")?


Центр - это центр, а край - это край. Если бы все было в одной точке, то эти вещи не назывались бы по-разному и никак не различались. Не было бы речи отдельно о горизонте и отдельно о сингулярности. Логично? Логично. К тому же, сингулярность - это не обязательно точка. Во вращающихся ЧД (а все реальные ЧД так или иначе вращаются) сингулярность представляет собой кольцо.
>>31197

>Нихуя подобного.


Твоя мамаша не шлюха, хочешь сказать? Нихуя подобного.
258 731215
>>31170
Какая потенциальная энергия на собирание всей массы в одном месте? Хуйни нагородил какой-то. Единственное реальное ограничение - это эддингтоновский предел. Если убрать его и представить, что во Вселенной имеется бесконечное количество вещества, то ЧД можно было бы бесконечно раскармливать до бесконечной массы (опять же при условии, если ты будешь намеренно вливать в ЧД многие миллионы и миллиарды масс Солнца вещества).
259 731217
>>31214

>Собственно, это и есть ЧД


Нет. ЧД - это объект в центре, сингулярность. Пространство между горизонтом событий и самой ЧД это не ЧД. Это по сути не видимое нам загоризонтное продолжение аккреационного диска.
260 731257
>>31203

>я про боинг над донбассом


Лол, что тебе с ним ещё непонтно-то в 2к22
261 731261
>>31217
Сам придумал "определение", сам же его и ввел? Ебать ты физик. А другие физики в курсе такого определения черной дыры? Или ты пока еще не отправил свою статью в нобелевский комитет?
262 731265
Что дешевле:
Изменить мясной мешок для работы в космосе.
Или робот с продвинутым ИИ?
263 731266
>>31265
зачем ии?
я думаю аватары - будущее.
просто гуманодиный робот с управлением из кресла. надевает ВР-хедсет, берешь джйостики -и вауля ты на марсе
264 731267
>>31266
Пинг слишком большой. Автономность нужна минимум на несколько минут.
В добавок биомеханика избыточна как механизм.
265 731312
>>31266

>я думаю аватары - будущее.


>просто гуманодиный робот с управлением из кресла


В зависимости от расположения Марса относительно Земли, время движения электромагнитного сигнала - от 3 минут 6 секунд до 22 минут 17 секунд до Марса и столько же обратно.
Пошевелил ты рукой робота, в аватар кресле и нихуя не произлшло, через 45 минут ты увидишь, что робот пошевелил рукой на марсе. Охуенное кино блять.
266 731336
>>31312
"Для работы с марсианским теледублем нужен настоящий тормоз, со скоростью реакции от 22 до 45 минут"
267 731373
>>31146
Что за человек, почему ты предыдущие даты умолчал?>>31312
>>31336
В контру не погонять, но в шахматишки рубануть самое то. Да и полуавтономным роботам давать указания бурить тот-то камень, фоткать ту-то какашку или лететь туда-сюда тоже можно.
Хотя стоп, они уже.
268 731377
Возможно ли увидеть звёзды днём? Отфильтровать синий цвет неба, сделать выдержку, усилить статичный свет?
Небо ярче, но свет звёзд же никуда не денется, верно?
269 731384
>>31377
Рассеяние света идет во всех длин волн.
Свет звезд никуда не девается, но он слишком зашумлен Солнцем.
Однако есть небольшие лазейки. Использовать ближний инфракрасный диапозон, точнее то что атмосфера пропускает.
Другое это использовать поляризаторы, рассеянный свет солнца в атмосфере поляризуется в зависимости от угла. Если по принципу адаптивной оптики управлять поляризаторами, то можно хорошенько затемить небо.
1642201390350.gif576 Кб, 640x358
270 731421
Анон внезапно оказывается на луне, которая является внутренней из 2 лун некоей планеты. Оказался анон на приливно-захваченной луне, притом на стороне внешней.
Для анона на поверхности этой луны эта информация НЕ известна.
Всё, что будет доступно анону, наблюдение за звёздами, местным солнцем и другой луной.
Вопрос: сможет ли анон понять, что находится на дальней стороны луны, или же будет думать, что оказался на планете и луна на небе - это собственная луна планеты на которую его занесло, а не внешная луна более большой планеты?
271 731427
>>28640 (OP)
Сап, Двач. Есть один вопрос.
Предположим, человек находиться на Луне, на её светлой стороне. С Луны ему видна Земля. Некие ксеносы бомбардируют Землю алмазными штырями, скорость каждого из которых - 75%С.
Что увидит наш гипотетически наблюдатель? Взрывы будут ему видны?
272 731448
>>31421
Сможет, но для этого нужно приложить усилия:
1. Интеллектуальные, т.е. замерить орбитальные параметры. Без приборов тяжеловато.
2. Брутфорс. Пройтись на другую сторону луны. Не требует никаких интеллектуальных усилий.

>>31427
Зависит от размера штырей и их количества (в сторону увеличения):
Не увидит ничего.
Увидит вспышки.
Увидит увеличение яркости Земли вплоть до солнечной и далее.
Не увидит ничего.
Алсо, не увидит ничего если светлая сторона не смотрит на Землю до тех пор пока мощность бомбежки не испепелит Луну.
273 731468
>>31448

>Алсо, не увидит ничего если светлая сторона не смотрит на Землю до тех пор пока мощность бомбежки не испепелит Луну.


Прошу прощения, я не совсем Вас понял...
274 731471
Если насобирать с кучи разных планет в галактике урана и навалить его в одно место запилив планету размером, скажем, с Уран, что будет?
Природный уран сможет в самоподдерживающуюся цепную реакцию? Будет ли эдакая звезда, но не на синтезе ядер, а на распаде?
Как запилить такую из подручных средств если такой вариант не загорится? Луна тория? Коричневый карлик ксенона?
Кстати, звезда из ксенона, загорится? Как будет выглядеть если да?
275 731474
>>31468
Если ты на светлой стороне Луны в то время как она между Землёй и Солнцем, то ты Землю не видишь.
16661835819830-wm.webm3,5 Мб, webm,
960x720, 0:30
276 731488
Когда там уже по земляшке въёбет что-нибудь мощное, километров 20 в диаметре? Всё равно человечество на грани вымирания, так хоть фейерверк 🎆 посмотрим. Что там моченые говорят?
277 731493
>>31147

>Есть сайт, где даются более приближенные к действительности оценки.


А есть сайт где рассказывают, что земля плоская. Нахуй ты высеры всяких уёбков за чистую монету принимаешь?

>Согласно им, метеорит был диаметром минимум 180м, массой более 1.5млн тонн


Кулстория нахуй, пешыти исчо.

>Каким образом ударная волна от взрыва мощностью 1.5 мегатонны, преодолев более 40 км в разреженной атмосфере


Обычным блять образом. Что бы ей помешало например? Жирная жопа твоей мамаши?

>смогла повредить столько сооружений


Сколько?

Стёкла, кое где старые гнилые двери повышибало, всё.
Полторы мегатонны всё таки.
278 731495
>>31147

>Каким образом ударная волна от взрыва мощностью 1.5 мегатонны, преодолев более 40 км в разреженной атмосфере (а эпицентр был на высоте более 20км) смогла повредить столько сооружений?


Направленность.

20км это не разреженная атмосфера, и тем более то что ниже. Разреженная (когда длина пробега молекулы превышает расстояние между ними, и способность проводить звук теряется) начинается на границе ионизации, т.е. ~80-90км. На 20км неебически плотный воздух, вполне способный передавать механические колебания, особенно такие энергетичные. Спускаемые аппараты (менее плотные чем камни) к той высоте уже давно перестают гореть и замедляются.
279 731501
>>31448

>Сможет, но для этого нужно приложить усилия:


>1. Интеллектуальные, т.е. замерить орбитальные параметры. Без приборов тяжеловато.


Тяжело, но не невозможно? Ага. И как без приборов всё это измерять? Как замерять движения небесных тел и время с точками движения?

>2. Брутфорс. Пройтись на другую сторону луны. Не требует никаких интеллектуальных усилий.


Это даже если вся планета ровная и гладкая, как шароид, займёт дни и недели. А если там рельеф разнороден, выступы, трещины, отвесы и впадины даже в 10м станут серьёзной проблемой
16421068926860.png1,9 Мб, 1098x2072
280 731502
Кстати, а почему в спейсаче нет - как в любом другом порядочно разделе - своего фап-треда?
Где мне дрочить на виды космоса из ИРЛ, 3д-рэндерингов и 2д-картин в представлении художника?
281 731504
>>31501

>Тяжело, но не невозможно? Ага.


Да.

>И как без приборов всё это измерять? Как замерять движения небесных тел и время с точками движения?


Тебе - никак, ты вторым методом пользуйся.

>Это даже если вся планета ровная и гладкая, как шароид, займёт дни и недели. А если там рельеф разнороден, выступы, трещины, отвесы и впадины даже в 10м станут серьёзной проблемой


Да, сложно. Но придумывать как орбиты замерять не нужно. Кандыхай на край света и через пару лет увидишь что к чему.
282 731505
>>31502
Как это нет фап-треда? Тут их много.
Нормисы фапают на негромексиканку в спасехтреде.
Извращи фапают на огуречих в кербалтреде.
Совсем извращи фапают на отечественную космонавтику в раскосом треде.
Астроаутисты фапают в пикчетреде.
Омичи в окб.
Я вот поехавший тута фапаю на твой вопрос.
283 731518
>>31471
Скорее всего снаружи будет твердая оболочка из раскаленного урана под ним мантия из урана расплавленного, в центре наверно твердое ядро.
284 731519
>>31518
Не взлетит загорится? Отстой пидорский. Что ещё эдакого можно отчебучить планетарного масштаба чтоб сработало?
285 731520
>>31471

> Если насобирать с кучи разных планет в галактике урана и навалить его в одно место запилив планету размером, скажем, с Уран, что будет?


ебанёт

> Природный уран сможет в самоподдерживающуюся цепную реакцию?


конечно, только за коэффициентом размножения никто не уследит

> Как запилить такую из подручных средств


никак, или реальные звёзды или реактор или ебанёт
287 731522
>>31521
Изображение открыть не могу, но скорее всего она в искусственных цветах как и большинство красивых фоточек туманностей. Или там розовые космические единороги? Тогда - чистая правда.
288 731524
>>31520
А если столько урана навалить чтобы сила отталкивания от "ебанет" не перестливала гравитацию? Посчитать массу такой йобы можно?
289 731525
>>31521
Что ты называешь обработкой?
Так-то в любой камере будет обработка в том или ином виде. Дебайеризация (для цветных матриц), гамма-коррекция, баланс белого - это лишь самый минимум от необходимого чтобы просто посмотреть картинку.
В любом случае, "обработки" там не больше чем в телефонах сейчас делают.
290 731526
Что такое Оумуамуа? Космисечский мусор?
291 731527
>>31526
Корабль пришельцев, о нем еще Кларк повесть написал.
292 731528
>>31527

>Кларк


Кент?
293 731529
>>31528
Кому-то наверно он и кентом был, но вобще-то Артур.
294 731530
>>31521
Любое астрофото с профессиональных инструментов, которое не ч/б это обработка, плюс глазом ты так никогда не увидишь.
JWST NIRCam затрагивает только совсем небольшой диапазон видимого спектра, красный цвет, и это фото - результат наложения шести ч/б фото, каждое с разным фильтром ( https://jwst-docs.stsci.edu/jwst-near-infrared-camera/nircam-instrumentation/nircam-filters ), конкретно тут использованы F090W, F187N, F200W, F335M, F444W, F470N.
Дальше какой-нибудь астроном берет эти фото, накладывает все это слоями в фотошопе и каждому назначает цвет, по принципу или чтоб было красиво, или чтоб выделить на фото какие-то нужные астрономам детали, конкретно тут сопоставление цветов и фильтров такое:
Purple: F090W
Blue: F187N
Cyan: F200W
Yellow: F335M
Orange: F444W
Red: F470N
Подробнее детали того, как делалось фото, можно посмотреть здесь: https://webbtelescope.org/contents/media/images/2022/052/01GF423GBQSK6ANC89NTFJW8VM
295 731532
>>31526
Разведывательный зонд, который при входе в систему собирает данные и передает их следующему зонду или станции, которые уже объедены в громадную разветвленную сетевую флотилию в межзвездной среде.
Скорее всего кусок льда(метан+азот), который выбросило в межзвездное пространство из протопланетного диска на стадии конденсации. Или какую-нибудь камешек, который пролетел через молекулярное облако и оброс "шубой". Такого говна в нашей галактике должно быть полно, тем более не далеко от областей звездобразования.
296 731533
>>31530

>Любое астрофото с профессиональных инструментов, которое не ч/б это обработка,


Нет. Вообще любое фото это обработка в том или ином виде. Даже если брать монохромную камеру от видеонаблюдения, то там будет как минимум гамма-коррекция, а реально больше всего.
297 731534
>>31527
>>31532
Единственный способ достоверно разобраться со всем этим - догнать, своими глазами посмотреть, пощупать и кусочек на память отломить.
298 731535
>>31526
Вполне возможно, что инопланетный зонд/космический корабль.
299 731536
>>31524
на массах от небольшого астероида до нейтронки ёбнет, при сборе НЗ тоже собственно ебанёт, но уже не из-за урана конкретно
300 731538
>>31533

> Вообще любое фото это обработка в том или ином виде.


Совершенно точная и абсолютно бесполезная информация, спасибо.
301 731547
Какие же людишки тупые, увидели космическую какаху и сразу начали строить влажные фантазии теории про инопланетных высокоразвитых господ, которым будто бы интересна какая-то там земляшка.
302 731548
>>31547
Рептилоид, мы все про вас уже знаем.
303 731553
>>31547
Антисетипетух, не трясись так.
304 731556
Какой щас самый мощный космический двигатель у человечества? До 20км/с разгонит хоть?
305 731557
>>31556
Юпитер, если тебя километры в секунду интересуют. А так скорость не в мощность двигателя упирается а в массу аппарата, запас топлива и удельный импульс, и тут вообще прожорливые ЖРД не в фаворе а модно-молодежно всякие ионные двигатели, кушающие ксенон по чайной ложке но зато с очень большой скоростью реактивной строи и, соответственно, удельным импульсом.
sage 306 731558
>>31504
Пошёл на хуй, уебан.
307 731561
>>31536
Блэт, то есть fission star никак не построить?

Рептилоид из расы 4 типа по Кардашян залетевший на земляшку на челябинском метеорите который есть посадочная капсула отстыковавшаяся от нума-нума омаймай омуамуа ну и название же вы дали, пиздос
308 731564
>>31557
Ясно, хуйня короче. Межзвёздные перелёты будут недоступны ещё тысячи лет с такими медленными темпами апгрейда движков.
А чё там с варпом?

Скорость света ближе, чем казалось: ученые создали первый пузырь Алькубьерре

Еще одна технология из Star Trek оказалась больше наукой, чем фантастикой. Ученые под финансированием DARPA создали так называемую «warp bubble» — область пространства, способную расширяться или уменьшаться быстрее скорости света. Об этом сообщает бывший специалист НАСА по варп-двигателям Гарольд Уайт.
309 731566
>>31564
До релятивистских скоростей вполне возможно разогнать через парус.
Тут идея проста. Ставим мощный лазер/ионную пушку и ебашим на зеркало/большой магнитный контур на аппарате. Светим несколько лет и он спокойно достигает долей скорости света.
Альтернатива с парусом запускаем аппарат ближе к солнцу, и там раскрываем парус. Скорости будут не релятивистские, но можно загнать быстрее, чем маневр у Юпитера.

Варп маняхуйня, человечеству будет проще создать ЧД планетарной массы, чем хоть какой-то варп драйв.
310 731575
>>28640 (OP)
Анон, откуда берется реликтовое излучение?
Да, я знаю, свет далеких галактик миллиарды лет до нас - вот это всё.
Но ведь большой взрыв произошел в точке. И от этой точки стало все разлетаться. И уже через 200 млн лет сформировались первые галактики. Логично предположить, что диаметр вселенной в этот момент был не более чем 200 млн световых лет радиусом. А это значит, что из этой сферы все излучение первичного взрыва улетучится за 400 млн лет. И после 400 млн лет никакого света испущенного в первые моменты в этой сфере, в которой находится вся материя, уже не будет.
Ну и как можно что-либо наблюдать спустя 17 000 млн лет?
311 731580
>>31575

> Логично предположить, что диаметр вселенной в этот момент был не более чем 200 млн световых лет


нет, предположение в корне неверно

> А это значит, что из этой сферы все излучение первичного взрыва улетучится за 400 млн лет.


нет, аналогично

> И уже через 200 млн лет сформировались первые галактики


и вообще нахуя ты взял эту дату, реликтовое излучение реликт 380к лет от БВ, образование галактик событие совершенно иного рода
312 731583
>>31561

> fission


это довольно комплексная хуйня с обратными связями с продуктами, сваливанием в кучу говен дающих нейтроны даже с идеальными говнами можно сделать разве что одноразовую хуитку максимум на несколько суток, причем разумеется не из урана, даже исключительно обедненный для такого он слишком активный
313 731589
>>28640 (OP)
На юго-западе Сириус примерно в 10 градусах над горизонтом.
ПОЧЕМУ ОН ТАК БЕШЕНО МЕРЦАЕТ
314 731590
>>31532
Тоже сначала думал, что это всего лишь камешек. Но когда "кмашек" при приближении к солнцу стал менять траекторию, при этом не оставляя за собой никакого следа из газа и пыли как это кометы делают, то появилась уйма вопросов.
И в это же время, все эти мочёные начали придумывать различные манятеории, почему он так движется и ускоряется как не должен был по всем расчётам и натягивать сову на глобус продолжают до сих пор. Ни одна из их манятеорий вменяемого подтверждения не получила.
Я вот ни разу не сторонник теорий заговоров, но в данном случае ребята прям ваще безпалевно хуйню на уши начали вешать.

Скорее да разведывательный зонд, скорость для камешка у него неебическая.
Но если какие-то пидарасы разведывают по всей галактике, то какого хуя они ничего не передали с помощью этого зонда?
what-the-fuck-am-i-reading.jpg63 Кб, 540x739
315 731595
>>31566

>человечеству будет проще создать ЧД планетарной массы


>ЧД планетарной массы

316 731599
>>31595
Планетарной в смысле Черные Дыры массой как у планеты. Начиная 10 в 22 степени килограммам и заканчивая 10 в 28 килограмм.
В природе нет механизмов образования таких объектов кроме первичных.
317 731600
>>31558

>Моментальный переход на оскорбления от тупого быдла


Типично.
318 731609
>>31595
таки шо вас смущает?
319 731613
>>31575

>Но ведь большой взрыв произошел в точке. И от этой точки стало все разлетаться.


Неверно, стало расширятся пространство и плотность вселенной начала падать. когда плотность уменьшилась настолько что вселенная стала прозрачной для света и свет стал испускаться из всех точек пространства. Можно сказать что пространство расширяется вместе с этими условными точками и свет от них будет доходить до нас, кроме света тех точек что движутся быстрее скорости света.
320 731615
>>31474
Откула тогда на Луне будет видно Землю?
321 731616
>>31615
со стороны, повернутой к Земле вестимо.
322 731619
>>31575

>откуда берется реликтовое излучение?


400 000 лет после Большого взрыва произошла рекомбинация - электроны зацепились за протоны, образовав атомы водорода и сделав вещество электрически нейтральным. После этого Вселенная стала прозрачной для излучения - фотоны получили возможность свободно летать в пространстве. Эти фотоны и есть космический микроволновый фон.

>Но ведь большой взрыв произошел в точке


Нет, Большой взрыв произошел во всем пространстве сразу. Никакой "точки" не было, у расширения Вселенной нет центра.
323 731620
>>31590
Есть такой мужик, зовут Авраам Леб, астрофизик, причем весьма маститый. Вот он первый в научной среде начал продвигать гипотезу, что Оумуамуа - это летательный аппарат ВЦ. Даже книгу про это написал. Разумеется, другие моченые сразу набросились на него, ибо слишком еретические мысли начал высказывать.
sage 324 731628
>>31600
Нет, тупое быдло и уебан здесь ты.
Вместо того, чтобы отвечать, выёбываешься.
Ты, мразота, в Треде Тупых Вопросов вместо того, чтобы отвечать на тупые в том числе вопросы, ты просто засераешь тред, оставляя ответ без ответа.

- Анон, а есть какие-нибудь формулы вычислить дельту при таких-то условиях?
- Да, есть)))
- ...
- )))
- Может тогда скажешь?
- Нахуя? Ты всё равно не поймёшь)))


И вот таким образом ты уже не один тред, а месяцов 6, не меньше, отвечаешь на разные вопросы. Это ты чтоли тот дегенерат, который в /б/ хвастался, что ходит по разделам и даёт ложные ответы, потешаясь с этого?
В любом случае поёшл нахуй илитку строить в какое-нибудь другое место.
325 731631
"скорость света" это скорость причинности(casuality)?

но скорость света в вакууме не равна скорости света в материи?
а скорость причинности что в ваккуме, что в ядре нейтронных звезд - одна и та же?

скорость причнинности это "скорость света в вакууме"? скорость распр-ия э.-м-ых волн вакууме?
326 731635
>>31631
У Пенроуза в своих работах вроде "Структура пространства-времени" и "спиноры и пространство-время" подробно и исчерпывающе рассматривает каузальность с геометрической точки зрения, и как это связанно со "скоростью" полей. Хотя это не научпоп и сразу в ебло тычат группами Ли.

>"скорость света" это скорость причинности(casuality)?


В целом да, но есть некоторые оговорки как с квантовой точки зрения, так с ОТОшной.

>но скорость света в вакууме не равна скорости света в материи?


типа того.

>а скорость причинности что в ваккуме, что в ядре нейтронных звезд - одна и та же?


Вот и тут появляются ньюансы квантовой магии.

>скорость причнинности это "скорость света в вакууме"? скорость распр-ия э.-м-ых волн вакууме?


Тут надо сказать так, если эм волны предоставлены только сами себе, то их скорость равна скорости причинности.
327 731645
Зачем нужно изучать космос, не ближний, а за пределами галактики, если то что мы там наблюдаем было миллионы лет назад. А покинуть пределы солнечной системы мы все равно не сможем
328 731649
>>31645
интересно жи.
ну и фундаментальная наука, ака попытки описания как оно все в мире работает, в целом полезная штука.
329 731657
>>31649
Что описать, что там такие звезды из водорода и гелия? Ну это же и так понятно, а черные дыры ещё 50 лет назад открыли, ничего с тех пор не изменилось
330 731660
>>31616
Простите за вопрос, но разве Луна повернута у Земле не светлой стороной?
331 731661
>>31657
под "звезды из водорода и гелия" лежат терабайты данных и стопицот теорий, чо ж там и как происходит с водородом и гелием
332 731662
>>31660
Луна повернута к Земле все время одной стороной. Но она сильно не всегда светлая.
image40 Кб, 600x300
333 731663
334 731664
Возраст:
Stephenson 2-18 - 14-20 млн.лет
VY Canis Majoris - 10 млн.лет
Betelgeuse - 10 млн. лет
Солнце - 4,6 сука, млрд. лет

Как так получается, что самая, сука, ничтожно мелкая звезда оказалась старше звёзд-переростков, которые уже скоро подохнут. По логике они должны быть на миллиарды лет старше Солнца, которому только предстоит стать красным гигантом.

Что случилось с ними?
335 731665
>>31664

>Как так получается, что самая, сука, ничтожно мелкая звезда оказалась старше звёзд-переростков, которые уже скоро подохнут.


Тот был умней, кто свой огонь берег,
И обогреть других уже не мог,
Но без потерь тадам-дадам
Дожил до теплых дней
336 731667
>>31645
Потому что интересно, блядь. Если тебе ничего дальше твоей обоссаной заплесневелой хрущевки не интересно, то это не значит, что другие люди мыслят так же.
337 731668
>>31664

>Как так получается, что самая, сука, ничтожно мелкая звезда оказалась старше звёзд-переростков


Чем менее массивна звезда, тем дольше она живет. Массивные горячие звезды живут по звездным меркам мало.
338 731669
>>31664

>Как так получается, что самая, сука, ничтожно мелкая звезда оказалась старше звёзд-переростков


Чем менее массивна звезда, тем дольше она живет. Массивные горячие звезды живут по звездным меркам мало.
231.png274 Кб, 800x429
339 731677
Messier 87 — сверхгигантская эллиптическая галактика.
Радиус: 60 000 световых лет

Messier 49 — эллиптическая галактика в созвездии Дева.
Радиус: 80 000 световых лет

Почему 87 сверхгигантская, а 49 нет, хотя первая меньше по размерам? Кривой перевод русской педевикии?
340 731679
>>31677

>Почему 87 сверхгигантская, а 49 нет, хотя первая меньше по размерам?


это не градация по размерам, а попадание под признаки классификации. Несколько ебануто, но вот так исторически сложилось.
341 731680
>>31679
Какими ебанутыми признаками квалифицируется сверхгигантность, если не размерами? В Последовательности Хаббла не нашёл
2084506900.jpg88 Кб, 897x900
342 731681
Кто нить пояснит мне за красные карлики и за их активность в плане вспышек которая может стерилизовать всю возможную жизнь на находящихся рядом экзопланетах?
Казалось бы красные карлики из-за низкой скорости термоядерного сгорания водорода имеют очень большую продолжительность жизни — от десятков миллиардов до десятков триллионов лет по сравнению с нашим Солнцем которое сжигает шустрей свой запас но и при этом вполне стабильна в плане вспышек, раз на земле за миллиарды лет развилась жизнь, почему красные карлики так активны?
343 731685
>>31681
В малых звездах вещество в среднем холодное. Оно по большей части представлено из нейтральных атомов и редких ионы. В таковом веществе передача тепла идет по сути только за счет конвекции. Эта конвекция всегда идет в турбулентном режиме. По всей толще звезды проходят вихри плазмы, которые отрываются от недр и медленно сплывают. Иногда вихри задерживаются в толще, слипаются в один гигавихр и всплывают разом, так получается супервспышка.
В средних и крупных звездах вещество в толще относительно горячее. Горячее настолько, что в нем преобладают ионы и давление изучения преобладает над давлением ионов. В таком состояние свойства поведение вещества по сути зависят от излучения. Так появляется зона лучистого переноса энергии. Она изолирует холодные внешнею область от недр и препятствует образованию больших вихрей и задержки их в глубине.
Хотя вихри все равно образуются, но они жиденькие и только у поверхности.
344 731689
>>31661
Так уже известно же что термоядерная реакция
>>31667
Что интересного маня?? Ты можешь ответить, там точно такие же звезды.
345 731692
>>31680
Видимо масса.
346 731699
Если эффект оберта зависит от гравитационного колодца и относительной скорости, значит ли это, что вне нашей галактики ракетные двигатели будут менее эффективны?
347 731700
>>31689

>Что интересного маня??


Интересно, как мир устроен, хуесоска барачная. Я говорю, если тебя не интересует ничто из находящегося за пределами твоего обоссаного барака, то это не означает, что все люди мыслят так же.
348 731729
>>31699
Да, но гравитационный колодец галактики связан с ее гало, а не с видимой частью галактикой. И опять эффективно будет относительно какого-нибудь войда и на таких масштабах уже сказывается расширение вселенной.
349 731730
Почему нельзя вместо большого и дорогого телескопа купить пятьдесят телефонов гэлэкси с22, примотать их скотчем к батарейкам, и выдать каждой роборуке, чтобы она умела крутиться для фокусировки?

Это выйдет явно дешевле, чем всякие бракованные джеймсы уебы.
350 731734
>>31730
нужна сплошная или совокупная площадь зеркала, а не просто большая база, как у интерферометров
351 731735
>>31734

> нужна сплошная или совокупная площадь зеркала, а не просто большая база, как у интерферометров


Можно отрегулировать все таким образом, чтобы каждый смарт снимал своей камерой определённую часть космоса, чтобы каждую фотку можно было склеить. Допустим пускай будет погрешность в 200-300 пикселей (по краям накладываются друг на друга), но это легко поправимо постобработкой.

А за счёт того что в телефонах установлен 5г модем, фотки будут быстро грузиться на земле, которые будет обрабатывать стенд из видеокарт нвидиа 4090.

Да я же гений нахуй.
352 731737
>>31730
Очень низкое угловое разрешение у твоей системы. Диаметр объектива камеры телефона около 1 см, угловое разрешение будет больше 10 угловых секунд для оптического диапозона. У Уэбба разрешение в оптическом диапозоне - это сотые доли угловой секунды. Именно это позволяет получить мелкие детали на изображении.
353 731738
>>31730
Очень низкое угловое разрешение у твоей системы. Диаметр объектива камеры телефона около 1 см, угловое разрешение будет больше 10 угловых секунд для оптического диапозона. У Уэбба разрешение в оптическом диапозоне - это сотые доли угловой секунды. Именно это позволяет получить мелкие детали на изображении.
354 731741
Удивительно, что люди привыкли верить, что мы одни во Вселенной. Они просто никогда до конца не понимали, насколько она невообразимо массивна на самом деле. И мы знаем только до горизонта частиц наблюдаемой вселенной, который может быть всего лишь каплей в океане ее полного размера.
355 731750
Мы обнаружили экзопланеты с океанами глубиной в сотни миль, а не с потешными лужами как на Земле, настолько глубокими, что вода в конечном итоге затвердевает, не замерзая из-за давления. Если хоть в одном из них есть жизнь, такие гигантские существа вполне могут быть правдоподобны!
356 731751
>>31741
Мы уже нашли ряд вещей, которые в этой вселенной небесконечны.
Глядишь, и вселенной найдем концы.
357 731753
>>31751

> Мы уже нашли ряд вещей, которые в этой вселенной небесконечны.


Например? - КОЛИЧЕСТВО ЕБЫРЕЙ ТВОЕЙ МАМАШИ АХАХАХАХА
358 731755
>>31628
Завали своё вонючее ебало, быдлоскот. Тебе не место среди умных людей, срыгнись танцульки в ПукПуке смотреть как вы это любите. Ишь чего удумала, скотина тупорылая, на доске астрономии срать!
359 731756
>>31735
Нах что-то запускать вообще? У тебя есть 4090, пусть сама рисует. Открываешь спесь энжин, выставляешь охуевшие настроечки "выше среднего" и выставляешь астрофото неслыханного качества.
360 731757
>>31735
пчел. Если совсем на пальцах - излучение от далекого объекта размазано по области, откуда ведется наблюдение, очень тонким слоем - закон обратных квадратов, все такое. Нужно собрать в кучку достаточно энергии для регистрации, интенсивность суть энергия, проходящая за единицу времени через единицу площади - так что есть два путя, увеличение времени экспонирования или увеличение площади, на которую падает поток. Как только ты подошел к технически разумному порогу длительности экспонирования, остается только увеличиватт площадь.
361 731762
>>31757

> Как только ты подошел к технически разумному порогу длительности экспонирования, остается только увеличиватт площадь.


Так все дело в том, что 50 самсунгов мало?

Ну можно тогда 150 ксяоми купить. Выйдет дешевле, но нужно будет рекламу отключить.
362 731763
>>31762

>Так все дело в том, что 50 самсунгов мало?


дело в том, что у них линзочки маленькие.
363 731767
Можно ли затвердировать газовый гигант, чтобы выгазовогигантироваться на него?
364 731768
>>31763
Не настолько маленькие, чтобы не сделать красивую фотку космоса
365 731769
>>31768
если тебе надо просто "красивую фотку космоса" - сойдет и ксяоми, хуле.
366 731770
>>31769

> если тебе надо просто "красивую фотку космоса" - сойдет и ксяоми, хуле.


А зачем ещё всякие большие телескопы в космос выводить?
367 731774
>>31770
Затем, что там атмосфера не мешает.
368 731775
>>31774

> Затем, что там атмосфера не мешает.


Ну так и ксяоми не будет мешать
369 731777
Если представить, что Млечный путь размером с мячик для гольфа (46мм), то край видимой Вселенной (~90 млрд. световых лет) будет на расстоянии ~40 км.
370 731781
>>31777
О, да вы из Америки!
371 731783
>>31677
а МП 100к св лет
372 731784
Как узнали возраст Солнца?
373 731788
>>31784
На глаз прикинули ну, это ответ почти без рофлов
374 731794
>>31784

> Как узнали возраст Солнца?


От пизды назвали, как обычно это бывает в новуке
375 731795
>>31784
Спектральный анализ и прочая химическая ебала.
https://habr.com/ru/post/415845/
376 731797
>>31784
прям точно - никак. Вообще говоря, текущие модели звездной эволюции суть обоснованные предположения.
image.png393 Кб, 720x400
377 731806
378 731822
>>31775
У ксяоми нет большого зеркала, способного собирать дохуя фотонов, нет чувствительности к другим диапазонам электромагнитного спектра.
>>31784
По его спектру.
379 731825
>>31822

>У ксяоми нет большого зеркала, способного собирать дохуя фотонов, нет чувствительности к другим диапазонам электромагнитного спектра.


А вдруг завтра Лэй Цзюнь запилит аналог FAST во славу Партии и своей корпорации и у щяоми появляется большое зеркало способное собирать дохуя фотонов и чувствительное к другим диапазонам электромагнитного спектра, что ты тогда будешь делать?
А теперь серьёзный тупой вопрос:
Почему богатеи-миллиардеры которым один хуй некуда бабло девать не вкладываются в термояды-циклотроны-телескопы?
Их тупых избалованных детишек никто и не вспомнит когда те окочурятся от передоза, а вот если на телескопе имени Безоса откроют расу рептилоидов, то [redacted] и его имя войдёт в историю, увековечится. Да и просто это хорошее дело.
380 731827
>>31825

>что ты тогда будешь делать


Нихуя, мне похуй.

>Почему богатеи-миллиардеры которым один хуй некуда бабло девать не вкладываются в термояды-циклотроны-телескопы?


Большие научные проекты типа того же JWST по большей части финансируются правительством. Вклад частных предпринимателей там не очень большой. Ну а вообще это вопрос не ко мне, я не миллиардер. Я в душе не ебу, что творится в черепной коробке ходячих денежных мешков.
381 731832
Радиация от Солнца и радиация от ядерного взрыва однохуйственна?
382 731833
>>31832
Хуй знает
383 731835
>>31832
Если ты про само ионизирующее излучение, то частично похоже, но разных сортов. В электромагнитном спектре, что ядерный взрыв, что Солнце это просто очень черные черные тела/горячие плазменные шары. Хотя ЯВ в пике горячее будет.
Однако Солнце люто ебашит протона и ядрами гелия в виде солнечного ветра. ЯВ не способен в такое без специальных ухищрений и то, эти прелести только в эпицентре взрыва.
Но при ЯВ способно создавать радиоактивное загрязнение продуктами распада. Солнце такого не производит.
384 731840
>>31832

>Радиация от Солнца и радиация от ядерного взрыва однохуйственна?



Нет. Радиация от бомбы - поток ик, видимого света, немного рентгена и гамма-излучения (т.е. электрически нейтральные фотоны) и если бомба еще и термоядерная, то поток нейтронов (без электрического заряда).
Радиация от Солнца - ик, свет, уф, мягонький рентген и поток заряженных частиц - протонов, ядер гелия и чутка электронов (все имеют электрический заряд).
385 731854
>>31840

>перечисляет виды излучения


>микроволны не упомянул


Почему?
386 731856
>>31832

>Радиация от Солнца и радиация от ядерного взрыва однохуйственна?


От Солнца летят в основном альфа и бета, от ядерного взрыва в основном гамма и нейтроны.
387 731872
>>31856
Альфа с бетой разве не выделяются при ядерном взрыве? Или ты про то что атмосфера мешат?
388 731878
>>31872
Ядерный взрыв, идет за счет цепной ядерной реакции деления. Соответственно при взрыве выделяется преимущественно осколки деления и нейтроны. Гамма фотоны прут за счет ядерных переходах в короткоживущих продуктах и активации ядер нейтронами.
Хотя сейчас в ЯО используются термоядерный бустер, хоть он и выдает альфа излучение, но главный продукт там нейтроны.

И альфа частицам мешает в первую очередь оболочка бомбы, потом уже среда.
389 731882
>>31878
Спасибо за пояснение.
390 731906
>>31631

> "скорость света" это скорость причинности(casuality)?


нет, это скорость с которой система изменит своё состояние относительно "причины", это может быть скорость как света, так и скорость голубиной почты или курьера
391 731913
Есть ненулевая вероятность, что Земля полая. Помнится когда-то думали, что атом заполнен веществом, а оказалось как оказалось. Сейм?
392 731915
>>31913

>Есть ненулевая вероятность, что Земля полая


Есть ненулевая вероятность, что ты долбоеб.
393 731916
>>31913
Да, 99.99999999999% пустого пространства.
Как и у тебя в черепе. Да и у всего из обычного вещества с атомами-протонами.
16657600525190.mp45,1 Мб, mp4,
1920x1080, 0:20
394 731945
Вспоминая физику, температура - скорость движения молекул.
Что означает температура Планка? Какая это скорость? Световая?
395 731947
>>31945
Не забивай голову всякой чушью
16664570381780.mp4538 Кб, mp4,
536x492, 0:03
396 731948
image.png206 Кб, 1480x1226
397 731950
>>31945
Нет. Скорость света недостижима для частиц с массой. Ты можешь накачивать энергию сколько угодно и никогда не достичь скорости света.
Когда накачаешь столько энергии что достигнешь планковской температуры - хуй знает что дальше будет, обычная физика дальше не работает и нужно матанить квантмех в этой области.
Предполагается что частицы настолько охуевают от энергии что становятся миниатюрными чёрными дырами и тут же испаряются. Дальнейшая накачка энергии делает их ещё большими чёрными дырами, охлаждая это ещё быстрее.
398 731967
Анон, у меня тут солнечное затемнение, а я не готов. Что делать?!
399 731968
>>31967
Покайся, ибо грядет.
image.png200 Кб, 900x473
400 731971
>>31967
А у меня пасмурно. Жми туда где всегда пасмурно и тебе не придется переживать о затмениях.
Жителям пасмурносрансков астрономия доступна только по интернетам.
image193 Кб, 1280x960
401 731975
>>31967
я тож не готов ибо пикрил, но немношк видно было сквозь облака
402 731976
Как образовывались сверхмассивные черные дыры?
403 731977
>>31976
Те что сегодня есть -выросли из семян, основные варианты:
- очень большие ЧД из звезд 3 населения, самых первых и очень массивных
- ЧД поменьше (или даже из первого пункта) в очень плотных звездных скоплениях, набравшие массу за счет столкновения с себе подобными
- прямой коллапс облаков газа
Потом все это жрало окружающий газ и звёзды, сталкивалось с другими ЧД, и опускалось в центр галактик. Понятно, что таких ЧД, кому повезло вырасти до размеров сверхмассивных, не очень много.
404 731980
Че это за рофлового решетки для гриля у Н1 и Востока, соединяющие ступени?
Вот ЭТА породия на ракету способна улететь в космос? Вот эти вот потешные столбики держат такую махину и не ломаются?
Кек лол.
405 731981
>>31980
Это решетчатая межступенчатая структура позволяющая обеспечить горячее разделение ступеней.
Это надёжнее, т.к. не требует двигателей осаждения топлива при разделении ступеней; тебе не надо тащить отдельные двигатели, что хорошо.
Титан-2 и Протон тоже горячее разделение использовали. https://youtu.be/PBkZNMM_IUg
406 731982
>>31981
Есть прецеденты, что эта хуйня ломалась?
407 731983
>>31981
Из чего ее делают, чтобы ее не распидорасило от нагрузки?
image.png389 Кб, 533x800
408 731984
>>31980

> потешные столбики


дурак потешный, у тебя бронежилеты, картон, самолеты, автомобили из таких же "столбиков" делаются. Погугли школьные эксперименты по физике с куриным яйцом в руке, стаканом воды, который стоит на весу на обычном листе бумаге, бумажными кольцами, которыми ломают палку. Там идею объяснят
409 731985
>>31984
Шухов, спок.
410 731987
>>31980

>Вот эти вот потешные столбики держат такую махину и не ломаются?


пчел
411 731988
почему оно не ломается?
412 731989
ПОЧЕМУ ОНО НЕ ЛОМАЕТСЯ???
413 731992
>>31981
Есть какая-нибудь годная статья (можно научную с матаном) о механизме разделения ступеней семейства Р-7.
Хочу почитать, как эта решетка крепится к третьей ступени.
414 731993
>>31992
раньше тут были пиздатые статьи обо всем http://www.kik-sssr.ru/
но у меня чего-то не открывается даже с прокси, только главная загружается
поем, посмотрю, че за хуйня
image.png85 Кб, 562x792
415 731994
>>31992
12 соединителей и 6 "замков".
Отстреливается так же как и другое разделение, даже проще, наверное.
Сделаны из люминя какого-нибудь, с учетом сопромата, достаточно чтобы вес верхней ступени и нагрузки держать.
Просто так не получилось нагуглить подробности как ты хочешь, но это не звучит как что-то особо интересное в контексте ракетной техники.
Для меня что-то "интересное и необычное" это спаржа, ближайшее к чему было на спейс шаттлах, несимметричность, которая была на 4 носителях (включая шаттл), системы аварийного спасения (которых на шаттлах не было, зато на союзах их аж две как оказалось), стартовые столы (у союза это подвес на точках крепления боковушек и поворотность самого стола (что потом убрали за ненадобностью)), и прочие уникальности.
416 731995
>>31988
>>31987
А чому должны?
417 732006
>>31976
Сначала коллапсировали огромные облака газа/сверхмассивные звезды в ЧД десятков и сотен тыс. масс Солнца, потом эти ЧД оседали в гравитационной яме в центре галактики. Через миллиарды лет за счет аккреции они отожрались до миллионов и миллиардов масс Солнца.
418 732010
>>31976
В научпопстатьях почему-то обычно пишут: А хуй его знает.
419 732028
Почему масса чд уменьшается, когда одна из частиц из пары частица-античастица падает за горизонт?
420 732029
>>32028
Потому что создание пары уменьшает энергию вакуума, уменьшая энергию ЧД, которая суть масса.
421 732031
>>32029
А какая связь между энергией вакуума и чд? Почему масса отъедается от чд, а не от мимо пролетающих звёзд?
422 732032
>>32031
Энергия вакуума это энергия пространства, в котором находятся окрестности ЧД, это очень искаженное пространство с высокими энергиями.
Казалось, это пространство искажается из-за ЧД и влияя на пространство на ЧД не повлиять, но обратная ситуация возможна - высасывая энергию из окрестностей лишаешь массы ЧД.
423 732038
>>32032

>Энергия вакуума это энергия пространства


Но пространство то общее на всю вселенную, почему энергия отбирается не у неё?
и почему не предполагается что с точки зрения стороннего наблюдателя система ЧД- пара частиц не н горизонте событий не замкнута, и для стороннего наблюдателя будет нарушение закона сохранения?
424 732041
>>32028
Потому что с точки зрения внешнего наблюдателя она обладает отрицательной массой.
425 732053
>>32041

>отрицательной массой.


Нихуя, рождается пара частица -античастица, масса у античастиц положительная.
426 732054
>>32038
Локальная энергия же. Появляющиеся частицы какбе выправляют искривленное пространство отбирая у него энергию.

>>32053

> масса у античастиц положительная.


А энергии у пространства становится меньше.
427 732056
Виртуальные частицы это от лукавого. Теория возмущений не применима для полей с высокой напряженностью (даже сранный протон) нельзя считать и совсем ломается для гравитационных.
428 732059
>>32056
Ты гей.
429 732060
>>32054

>А энергии у пространства становится меньше


На массу двух рождённых частиц. Потом одна улетела- и рождает искривление на свою свою массу вокруг себя, а вторая упала на чд и чд стала на массу этой частицы тяжелее, следовательно искривлять пространство тоже стала сильнее.
430 732061
Кстати, а если виртуальные частицы рождаются под горизонтом, но не внутри сингулярности, с ними что происходит?
image.png6 Кб, 440x236
431 732065
>>32060
А по совокупности пространство потеряло энергии на массу этих двух частиц.
Это если они просто разлетятся.
В итоге у черной дыры теряется энергия на 2 частицы, обретается масса на две частицы, одна частица падает в чд, другая съебывает в пердя, оставляя потерю энергии (массы) на одну частицу.
432 732074
>>32061
Все что под горизонтом, падает в сингулярность, причем за конечное время. Даже небо, даже Аллах.
433 732075
>>32074
не обязательно, может шароебиться по орбите вокруг. Горизонт событий - это где вторая космическая больше c, а не первая.
434 732078
>>32074
Масса чд при этом вырастет?
image.png566 Кб, 718x797
435 732079
Я тут подумал: вот отправили меня на Плутон, дали шубу и сапоги и т.д., и вот я всаживаюсь на поверхность, включаю фонарь карманный бытовой походный обыкновенный и...
Превратится ли луч от фонаря в выжигающий все и вся луч смерти? Ведь фотон несёт не только свет, но и энергию, а значит что куда бы я не посветил, я туда же направлю потоки энергии и - соответственно - совершу нагрев освещённой зоны передачей тепла. А так как грунт Плутона есть замороженный газ и вода, то свет будет плавить и выжигать лёд превращая его тут же в газы?
436 732080
>>32075
Горизонт событий не космическая скорость по ньютоновской аналогии, а область из под которой нельзя вывести наружу геодезическую. Это имеет более серьёзные последствия.
Под горизонтом в ЧД даже в экзотических решениях все геодезические сходятся в сингулярности. Однако геодезические соответствуют инерциальному движению и можно наебать систему двигаясь с ускорением. Но рано топливо для ускорения у тебя кончится и ты упадешь в сингулярность. Тлен и безысходность.
Кстати, помимо самой гравитационной сингулярности, во вращающихся ЧД есть другие виды сингулярности порождаемые внутренними неустойчивостями, но не такие ультимативные как основная, но любые поля основательно ломают.
437 732081
>>32078
У ЧД нет как таковой массы.
Есть внешнее проявление которое можно интерпретировать как массу и в пределе это все что "опечатано" на горизонте. Все что под происходит горизонтом навсегда отрезано для вселенной.
image.png1 Мб, 1280x720
438 732084
>>32079
Ну, во-первых, ты задохнёшься, но это классика, просто хотелось упомянуть.
На Плутоне светлее чем в Питере, так что твой жалкий фонарик там будет как комариный пук в атмосферу.
Может если ты мегалюменовый фонарик типа пикрила включишь, то да, в паре метров какие-нибудь волатильные газы вскипятишь чутка, потом у твоего фонарика сядет батарейка или вскипят диоды.
Давай не забывать про такую вещь как энергия, там от Солнца примерно ватт на квадратный метр падает. А квадратных метров там почти как в России.
Единственный шанс как-то глобально повлиять на Плутон с помощью фонарика это иметь фонарик из антиматерии, притом с пикрила, а не карманный.
439 732094
>>32084
То есть, мало того, что Россия больше Плутона, РФ ещё и темнее этой маленькой планты, находящейся в 40 раз дальше от Солнца, чем Земля?
Да ты шутишь!
440 732105
Там Сурдин говорит, бетельгейзе опять потускнела. Поясните если она таки пизданёт, надо ли как-то защищать пека, ну там какой метал брать лучше и где, чтоб диски спрятать.
2.jpg55 Кб, 900x1200
441 732108
>>32079

>а значит что куда бы я не посветил, я туда же направлю потоки энергии


>А так как грунт Плутона есть замороженный газ и вода, то свет будет плавить и выжигать лёд превращая его тут же в газы?



У меня сосед, Санёк (тополиный пенёк), с IQ примерно 50-55. Решил прошлой зимой, после снегопада, что сугроб снега перед его крыльцом слишком заёбно и долго перекидывать дальше. Достал паяльную лампу из сарая (пикрил) и сказал, что щас по бырому растопит его нахуй. Три часа я наблюдал из окна, как он бегал с этой лампой вдоль сугроба. По итогу видимых изменений не произошло, пару сантиметров снега может растаяло и всё. И это при -15 где-то было, не так уж и холодно.
Сань, это ты? залогинься. :))
442 732110
>>31989
Ну вон прямо на твоей фото, кольчуга вполне себе сломалась.
Всё из тобой запощенного ломается. Кевлар крепкий, но не до бесконечности, вполне себе рвётся, даже не от самых мощных пуль, потому его много слоёв хуярят в броники. Газоблок, ну ваще смех, молотком разбивается легче чем кирпич.
Металлокаркас складывается охуенно, когда инжернер проектировщик хуесос с купленным дипломом не учитывает что в раше зимой снег и что с вон такой как раз плоской крыши он нихуя не скатывается.
ВСЁ в этом мире разрушаемо и ломаемо.
443 732111
>>31982
Нет. А чему там ломаться, металлическому профилю? Это элементарно рассчитывается, сопромат для начинающих студентов блять.
Дано нагрузка и параметры металлопрофиля, всё.
444 732134
>>32105
Потускнение Бетельгейзе вызвано в первую очередь тем, что она звезда не на главной последовательности и часто пердит газом и пылью массой итоговой массой примерно в земную.
Это никак не говорит о том сколько звезде осталось.
Даже если и ебнет, твоей пеке ничего не грозит от нее.
445 732146
>>32053
Для внешнего наблюдателя вторая частица - та, которая под горизонтом - имеет отрицательную массу. Иди читай про излучение Хокинга и испарение черных дыр.
446 732147
>>32056
Антисетипетух, виртуальные частицы проявляют себя в реальных физических явлениях, таких как эффект Казимира и лэмбовский сдвиг.
447 732148
>>32061
Падают на сингулярность, как и все что попадает под горизонт.
448 732163
>>32147
Чини детектор.
Виртуальные частицы это издержки теории возмущений, суть которого не в прямом честном квантование полей, а использование последовательных линейных приближений, которые по отдельности легко считать. Нулевое приближение соответствует "истинным" частицами, а с остальное виртуальные. Причем у виртуальность есть свой порядок виртуальности.
У метода возмущений есть целый класс неразрешенных проблем связанный с расходимостью рядов, а для некоторых полей(как кварк-глюонные) нет даже способа нормальной оценки.
Всякие эффекты где какбы проявляются виртуальные частицы считались через методы теорий возмущений и унаследовали у них эту хуиту.
449 732166
>>32163
Высрал ебаную шизофазию и думаешь, что типа умный? Пиздец ты дебил. Еще раз, гугли эффект Казимира и лэмбовский сдвиг. Это эффекты, напрямую вытекающие из существования виртуальных частиц.
Лучше занимайся самообразованием, а не пытайся отстаивать свое невежество и тупость с помощью уебищной демагогии.
450 732176
>>32147
>>32166
Я бы тебе ебало сломал за такой базар, туфлями дырявыми с секонд-хэнда за 500 рублей, в которых кто-то умер. Забил бы тебя нахуй, чтоб ты сука хавал с капельницы мышь горбатая.
451 732182
>>32176
Ебать, ты че так порвалась, чуханка? Хуев надроченных пересосала, перевозбудилась? Вытри сперму, выпей холодного молочка - остудись.
452 732194
Бывает ли молекулярная плазма?
Плазма из воды, скажем?
453 732195
>>28752

>с 50км наберет скорость от 0 до 1000 м/с. И эту тысячу нужно будет погасить, можно это сделать одним заходом перед посадкой или постепенно, не давая слишком сильно разгоняться при падении.


Че, блять? Про сопротивление воздуха слышал? Ступени нужно погасить только предельную скорость падения. Для человека это 200 км/ч, не думаю что для бустера это число сильно отличается.
454 732196
>>32195

>Для человека это 200 км/ч, не думаю что для бустера это число сильно отличается.


Напрасно не думаешь. Ступень флакона достигает 310м/с в свободном падении (до включения движков на посадку), а это больше тысячи километров в час.
455 732197
>>32195
парашютисты до 500 км/ч разгоняются без крыльев, а при устойчивом положении головой вниз можно 300 км/ч выжать. тот же гугл.
456 732198
>>32196

>Напрасно не думаешь. Ступень флакона достигает 310м/с


Хуя, ну ладно. Думал раз она такая огромная, то и сопротивление у нее будет высокое.
>>32197

>парашютисты до 500 км/ч разгоняются без крыльев


Ну, это на большой высоте.

>300 км/ч


Во это более реально, хотя мой гугл говорит про 240-250 км/ч
457 732206
>>32194
Вполне. Только это будет не молекулы, а ионы и радикалы. Такая плазма в ионосфере Земли присутствует и как раз по большей части из воды(протоны и гидроксилы).
458 732229
>>28640 (OP)

>тупых вопросов



1. Может ли быть сделан аппарат, способный приблизиться к газовой планете(например юпитеру) достаточно близко, чтоб ВСОСАТЬ образцы газа в себя и при этом вернутся обратно в целости и целомудрии?

2. Будет ли он представлять опасность на земле после возвращения?

3. Может ли спейс аппарат облучится так сильно, что это повлияет на землю после возвращения?

4. Газовые планеты обладают практической ценностью как источники топлива/энергии?
В варп слетать хватит?
459 732235
>>32229

>В варп слетать хватит?


Нет, не хватит. Варп - это вообще фэнтезийная хуета.
460 732239
>>32229
1. Вполне возможен. Но у планет гигантов состав атмосферы здорово меняется с высотой. По крайне мере выше аммиачно метановых облаков можно сделать несколько орбитальных витков, один фиг там водород и гелий.
2. Если там не ядерная силовая установка, которые въебет при приземление в жилой сектор, то нет.
3. Если не брать шизу макак на земле, то особо никак. Ну в теории аппарат может проболтался лярд лет и обрасти массой до уровня астероида за счет застевания в нем частиц, но земля к этому времени кончится уже.
4. На них гравитационные маневры удобны, ну еще потенциальные источник водорода для рабочего тела.
Забудь про варп, это маняхуйня.
461 732240
>>32229

> 1. Может ли быть сделан аппарат, способный приблизиться к газовой планете(например юпитеру) достаточно близко, чтоб ВСОСАТЬ образцы газа в себя и при этом вернутся обратно в целости и целомудрии?


Можно пройти по краю атмосферы и попытаться там что-то поймать в баночку, но на большой скорости, я боюсь, удастся поймать только чуть-чуть плазмы, которая мгновенно улетучится прежде чем ты успеешь закрыть крышку. Опустить что-то в атмосферу Юпитера и потом взлететь оттуда - что-то из области фантастики пока, нету таких двигателей. Плюс потом ещё надо с орбиты Юпитера (или даже с пролётной траектории) как-то вернуться назад к Земле, одно это потребует поряка 20+ км/с характеричтической скорости

> 2. Будет ли он представлять опасность на земле после возвращения?


Ну, придется обращаться как с радиоактивными материалами, но это мы умеем, этот пункт самый простой

> 3. Может ли спейс аппарат облучится так сильно, что это повлияет на землю после возвращения?


Как именно? Люди сейчас оперируют тоннами/десятками тонн сильно радиоактивных (делящихся, причем) материалов, компактно сосредоточенных в одном месте, сложно представить аппарат, который будет хуже этого.

> 4. Газовые планеты обладают практической ценностью как источники топлива/энергии?


Наса уже почти надуло пузарь Альбукерке, как раз надо один Юпитер целиком аннигилировать.

А если чуть серьезнее и ближе к реальности, то атмосферы планет-гигантов энергетической ценностью обладают только в контексте сжигания тамошнего водорода (или там метана с Урана) прямо на месте с хз чем в качестве окислителя (ну или в термоядерных реакторах, но там столько не нужно). Поднять что-то из атмосферы на орбиту будет стоить столько, что никто этим никогда не будет заниматься на химических двигателях, а если у нас будут двигатели на иных принципах и при этом с достаточно ысокой тягой, смысл возить водород ставится под вопрос, куда его столько надо?
462 732242
>>32229

>1. Может ли быть сделан аппарат, способный приблизиться к газовой планете(например юпитеру) достаточно близко, чтоб ВСОСАТЬ образцы газа в себя и при этом вернутся обратно в целости и целомудрии?


Да.

>2. Будет ли он представлять опасность на земле после возвращения?


Нет, если правильно сделан.

>3. Может ли спейс аппарат облучится так сильно, что это повлияет на землю после возвращения?


Нет, на землю не повлияет.

>4. Газовые планеты обладают практической ценностью как источники топлива/энергии?


Сами - вряд ли, окрестности - да.
Антивещество в магнитных полях, вода и азотности из спутников.

>В варп слетать хватит?


Пока нихуя не ясно как варп запиливать - нет.
image.png59 Кб, 239x211
463 732243
>>32240

>Наса уже почти надуло пузарь Альбукерке, как раз надо один Юпитер целиком аннигилировать.


А где купить антиюпитер?
На алиэкспрессе нет.
464 732287
>>32108

> У меня сосед, Санёк (тополиный пенёк), с IQ примерно 50-55. Решил прошлой зимой, после снегопада, что сугроб снега перед его крыльцом слишком заёбно и долго перекидывать дальше. Достал паяльную лампу из сарая (пикрил) и сказал, что щас по бырому растопит его нахуй. Три часа я наблюдал из окна, как он бегал с этой лампой вдоль сугроба. По итогу видимых изменений не произошло, пару сантиметров снега может растаяло и всё. И это при -15 где-то было, не так уж и холодно.


Орнул. Там лёд образовался и ему вместо чистки снега пришлось долбить его?
465 732302
>>32287
Я думаю, что это просто очередной случай, когда в голову приходит "гениальная" идея "ПОЧЕМУ БЫ ПРОСТО НЕ РАСПЛАВИТЬ СНЕГ ВМЕСТО ТОГО ЧТОБЫ ЕГО УБИРАТЬ", и эта идея получает моментальную реализацию, вместо того чтобы попытаться дать на нее ответ. А дать на нее ответ - это как раз тот случай, когда школьная программа физики пригождается по жизни.
У воды огромная теплоемкость (4.2 Дж/г/К), у льда тоже немаленькая (2 Дж/г/К), 2кДж/г на испарение (если не хочешь оставлять лужу которая льдом тебе останется), плюс плохая теплопроводность снега — вот и ответ, почему зимой не плавят/испаряют снег: энергетически на порядки проще его подвинуть.
Скажем, у тебя 10 квадратных метров которые ты хочешь держать чистыми, допустим, зимним утречком в -10℃ выпал свежий снег (плотность свежего снега примерно 100кг/м³), слой в 10см. Это 0.1м³, т.е. 10 килограмм.
0,2 МДж на нагрев снега + 3.33 МДж на плавление + 4.2 МДж на нагрев воды + 20 МДж на испарение = 27.7 МДж.
Это примерно энергия, которую можно получить при сжигании литра бензина. Но эту энергию ты не сможешь 100% эффективно потратить на кипячение снега на своем тротуаре, ты будешь бороться с теплопроводностью снега, нагревать горелку, тротуар,
небо, прах Клайва Томбо.

Тем временем, если ты соберешься перекидать 10 килограмм снега, то потратишь считанные килоджоули:
https://www.ccohs.ca/oshanswers/ergonomics/snow_shovelling.html
https://captaincalculator.com/health/calorie/calories-burned-shoveling-snow-calculator/

Relevant xkcd:
https://what-if.xkcd.com/130/
image.png645 Кб, 736x552
466 732307
Посоны, предлагаю вместо сжигания угля в паровозе поставить там атомный реактор для кипячения воды.
Можно даже на Марсе кататься, воздух для сжигания не нужен же.
Как вам такая экологически чистая идея? Какие подводные?
467 732315
>>32307
Говно идея - на Марсе рельсов нет.
image.png474 Кб, 640x480
468 732316
>>32315
Гогно вопрос, поменять колесные пары на колеса здорового человека и кататься без рельс.
469 732323
Мне когда-то скидывали ссылку на Википедию,
на некие математические формулы.

Это был ответ на мой вопрос - почему в безграничном Космосе нет места генерации бреда (пример: планета 1 в 1 как наша, только у диктора в прямом эфире вырос хуй на лбу с крыльями бабочки, оторвался и улетел в окно).

Напомните, что за формулы там.
470 732326
>>32307
Несколько вопросов. Как будешь охлаждать воду? Как будешь контролировать реакцию в реакторе? Ведь при остановке поезда реактор нельзя заглушить, а потом просто завести, поэтому придется куда-то девать избытки этой энергии.
471 732327
>>32326

>Как будешь охлаждать воду?


Радиаторы по всему составу.

>Как будешь контролировать реакцию в реакторе? Ведь при остановке поезда реактор нельзя заглушить, а потом просто завести, поэтому придется куда-то девать избытки этой энергии.


А пох, все время на полную 60 мегаватт пусть хуячит, мне не жалко. Как встанет где - может запитывать чего-нибудь.
А если утечет - нехай течет в реголит, один хуй экологии нет.
472 732328
>>32327

>Радиаторы по всему составу.


Не хватит для отвода тепла.

>А пох, все время на полную 60 мегаватт пусть хуячит, мне не жалко.


Нет не похуй, при простое эта энергия никуда не исчезает и ее нужно утилизировать, иначе пизда твоему реактору.
Если хочешь поезда на Марсе, то легче построить стационарную ЯЭС и питать рельсы электропоезда.
473 732329
>>32328
А ну там скажем плавильню с собой возить и электроэнергией реголит перерабатывать когда хочется, а тепло сбрасывать нагревая шлак тепловыми насосами. Как тебе такое, Милон Аск?
474 732330
>>32329
Нахрена тебе город на колесах, лол?
image.png51 Кб, 222x227
475 732331
>>32330
Чтоб все охуели как я могу а Илон не может, ему бы только твиторы покупать, лишь бы на марс не лететь.
image.png1,3 Мб, 1280x720
476 732332
>>32330

>город на колесах


Какие профиты, какие недостатки?
Преимущества перевешивают?
На сраной земляшке же строили подобную йобу, на марсе еще больше можно из-за гравитации.
image.png177 Кб, 446x220
477 732333
>>32331
Прилетают американцы на Марс, видят Битардск на рельсах и охуевают.
Screenshot2022-10-29-20-48-05-49948cd9899890cbd5c2798760b2b95377.jpg529 Кб, 1080x2400
478 732334
Пикрел это бан или что за хуйня?
Если бан, то за что? Я ничего не писал в комментах.
image.png234 Кб, 480x334
479 732336
>>32334

>сразу же порашный пост вместо астрономии


Я бы забил хуй на этот канал, на хуй тебе надо такое счастье?
Вон, Скотта "Наше Всё" Мэнли смотри, он в порашу не скатывается.
480 732337
>>32336
Я давно смотрю и читаю этот канал. На порашу абсолютно похуй, я как бы тоже либераха и видал хуйло в петле. Мне прежде всего интересны статьи, переводы на русский. Фильм о Кассини смотрел и другие. Теперь только Ютуб остался. Очень жаль, других подобных каналов на русском языке я не видел.
image.png166 Кб, 320x247
481 732338
>>32337
На русском языке если что и выходит годного, то это уже вторичное и много раз было в англонетах, и проникло к нам осмосом.
Потому давныым давно забил хуй на рунет, тут нечего ловить кроме двача потому что я говноед анало говнет, на форче нет общения
482 732343
>>32338

> рунет, тут нечего ловить


Почему так?
Русским языком так или иначе пользуется несколько сотен миллионов человек.
image.png58 Кб, 217x232
483 732344
>>32343
Английским на порядок больше, так что в численном выражении и годноты больше, и шансов на прорывность.
Взять ту же астрономию. У нас полтора инвалида с телескопами, шарящих астроёбов по пальцам пересчитать, там же такое население будет в среднем городишке техасщины.
484 732345
>>32344
Но почему?
У меня есть телескоп, но мелкий совсем, 114мм апертурой. Хотел бы купить больше, но они стоят бешеных денег.
Там что каждый второй себе с зарплаты может купить 200+мм?
MV5BZTBiYzYzZTMtZDg5Yi00ODJlLTg2ZWItMzBiZTg2MzI0Mjg2XkEyXkFqcGdeQXVyMTUyNzcwODk3.V1FMjpgUX1000.jpg136 Кб, 1000x1501
485 732346
Вот все говорят "скоро Йеллоустоунский супервулкан ебнет, всех лавою затопит, а кого не затопит лавой, того дымом задушит", а все потому что там лавы дохуя копится.
А я такой: а че бы не пробурить там и не спустить лаву в сторонку, давление стравить чутка?
486 732347
>>32346

>Вот все говорят "скоро Йеллоустоунский супервулкан ебнет


Это фантазии пидорашек, к реальности не имеет отношения.
image.png116 Кб, 400x388
487 732348
>>32345
Не только и не столько.
Там еще всякие Карлы Саганы и Фейнманы с Тайсонами промывали детишек, кружки полноценные были, наса популяризацией вовсю шпыняла.
А у нас что? В обычных школах вроде вообще астрономии нету на хуй, в илитной гимназии была, но никаких выездов не делали, популяризация науки в виде Дроздова и Очумелых Ручек не очень способствовала широкому кругозору и развитию интересов у широких масс.
488 732349
>>32347
Почему же тогда столько пиндосских фильмов про это? Росдеп платит?
489 732350
>>32349
Хуй знает, я не слежу за пиндоскими фильмами. Вообще там много желтушного говна, не стоит уделять этому внимание.
490 732351
>>32350
А вообще просто вулкан стравить не вариант? Ну там Везувий вдруг Помпейи 2.0 решит запилить, а ты знаешь об этом и ему лаву слил, и он такой "пук пук ой", и помпейи целы и волки сыты.
491 732352
>>32348

>А у нас что?


У нас всем похуй.
Пыне главное усидеть на троне;
Верхушке главное попилить бабла на яхту и виллу;
Верхушке поменьше - попилить бабла на мерседес и чтобы не выперли;
Обычным учителям и преподам - главное отработать смену и ебись оно конем, т.к. за копейки нахуй не надо че-то там популяризировать. Идейных единицы, но их выкидывает "верхушка поменьше".

В общем всё изначально идет с пыни и с его "группы поддержки". Сгнила голова - гниет и остальная рыба.
492 732369
>>32323
Космос безграничен, а не бесконечен. Описанная тобой хуета имеет ничтожную вероятность реализации, если в принципе возможна. Хотя учитывая, что Вселенная не бесконечная, планеты 1 в 1 как Земля с примерно теми же событиями в принципе не должно существовать. Подобное возможно лишь в рамках мультивселенной, которая вполне может быть бесконечной.
493 732374
>>32369

>Вселенная не бесконечная


Это тебе кто такое сказал?
494 732378
8 октября взлетел длинный марш 2д.
Почему он рассыпается на старте? Что валится с него?
У меня шаттл умер от этой хуйни!
495 732379
>>32374
Факты.
496 732385
>>32378
https://youtu.be/Fn-2Fd0e2RY
даже Скотт не знает, что это за хуйня точно. Физически защищает или таки от температуры.
.jpg77 Кб, 900x900
497 732413
>>32378
>>32385
Теплоизоляция обтекателя, чтоб не простудился холодной ночью. У Протонов стеганая одеялка была, например.
image.png942 Кб, 900x600
498 732434
>>32413

>У Протонов стеганая одеялка была, например.


Я помню эту стеганую одеялку, но я не помню чтобы он с нею взлетал и тем более чтобы она осыпалась с нее на старте…
499 732435
>>32434
Ну так с него снимали. А у них сделано чтоб осыпалось.
500 732436
>>32435
Какие они смелые.
Ладно хоть не осыпается когда огурцов запу-… Вейт…
501 732438
>>32307
На Марсе даже реактор не будет нужен, вода и так закипит.
502 732440
>>32436
Так у шаттла-то водородиум в баке, и там лютая пенища ебать, чтобы не испарился раньше времени.

И ладно еще отваливаются, у Дельты 4 изоляция эта вообще ГОРИТ на старте, причем это максимум страшно выглядит, словно вот-вот пизданет. И это у них нормально, ебанутые блять.
image.png292 Кб, 800x445
503 732442
>>32440
Знаешь, проломленный падающей хуйней бак гептила на Продолжительном Марте 2Ф тоже не очень приятные последствия сулит.
504 732449
>>32346
Маленький Принц легко мог прочищать вулканы ершиком ибо планетка у него была мелкая и вулканы по колено. С Йелоустоном такой трюк не пройдет, он зело велик.
505 732466
Почему люди планируют и мечтают терраформировать марс, но не хотят начать с ''терраформирования'' самой земели? Безжизненная пустыня сахара например та же. В чом выгода от траты земных ресурсов на благоухаживание пустых, облученных и холодных пустынь?
506 732468
>>32466
О терраформинге сахар и тундр тоже говорят. Как и в случае с марсом дальше разговоров ничего не заходит.
507 732469
>>32466

>Почему люди планируют и мечтают терраформировать марс


Бессмысленный пиздеж богатых долбоебов, которые хотят попиариться на популярной среди хомячков теме.
508 732476
>>32466

>Почему люди планируют и мечтают терраформировать марс, но не хотят начать с ''терраформирования'' самой земели?


Ну в далёком будущем сделают и на земле и на марсе, но ооочень не скоро. Если конечно разбертуся с проблемами на земляшке и в своих головах.
А так да, как анон выше написал, щас богатеи пиарятся, наивные аутисты типа Маска думают, что да реально лет через сто. Но нихуя нет, к сожалению. Лет 100-200-300 только проблемы с ебучей экономикой земляшки будут решать, с энергетикой, с ебланами с ядерным оружием, с болезнями и прочими хуями верчёными.

Ну и то что хотя бы мечтают, это уже хорошо. Всегда находятся те, кто мечты воплощает в реальность рано или поздно.
5073783f98008b66af46b0cb16c8d4f1.jpg19 Кб, 275x300
509 732477
>>32476
P.S. а чего переката нет?
Стикер255 Кб, 500x500
510 732483
>>32469

>богатых


Не неси хуйню, я вот бедный долбоёб, а тоже бессмысленно пизжу о колонизации Марса.
511 732489
>>32469
Скорее люди просто фантастики про яблони на Марсе обчитались, а яблони в Сахаре это не так интересно. Ну и кок-пок бэкап, кудах.
512 732516
>>32483
Твой пиздеж ни у кого не на слуху, о твоем существовании почти никто не знает. А вот о пиздеже какого-нибудь Маска знают очень многие. Про это шла речь.
513 732965
Как должна быть расположена планета, находящаяся между двумя звёздами парной звезды на относительно близкой и относительно круговой взаимной орбите, чтобы при определённых условиях планету, находящуюся в центре вращения этих двух звёзд, можно было там расположить стабильно и чтобы можно было бы считать географическим центром такой системы?
image.png254 Кб, 1116x686
514 732968
>>32965
Между двумя звездами она не может быть на взаимной орбите (если я правильно понял твой пост - в зоне влияния обеих звезд).
Она либо S-типа, т.е. на орбите одной звезды, либо P-типа, т.е. вокруг барицентра обеих звезд за сферами влияния каждой.
Между звездами вокруг барицентра стабильной орбиты нет.
515 732985
>>32968

>Между звездами вокруг барицентра стабильной орбиты нет.


Вообще ни при каких обстоятельствах? Неужели не подобрать комбинацию P-типа, которая будет не во внешней границе от звёзд, а во внутренней, настолько близко к барицентру, настолько это возможно ну или в настолько необычных условиях, при которых мы бы могли назвать ситуацию иллюзией положения в барицентре?
image.png154 Кб, 478x268
516 732986
>>32985
Вообще никак. Может быть, в идеализированной самостабилизирующейся симуляции из только трех неменяющихся тел.
517 732990
>>32968
Так, стоп.
На пике слева в двойной системе показана планета на P-орбите, охватывающей обе звезды. И как такую планету классифицировать?
Допустим у нас вся звёздная система на пике, это Звезда Петровича, которая только при сильном увеличении - как оказалось - представляет из себя пару двойных звёзд.
Как классифицировать эти звёзды и планеты?
Справа Альфа Петровича А (центровая) и Альфа Петровича Б (орбитальная), а слева Бета Петровича А (жёлтая) и Бета Петровича Б (оранжевая)?
Как тогда обозначать планеты? Я понимаю, как обозначают планеты той или иной одиночной звезды с классической центроориентированной орбитой планет, а тут-то как?
518 733009
>>32990

>Как классифицировать эти звёзды и планеты?


Ты верно подметил, что это проблема, на это обратили внимание:
https://en.wikipedia.org/wiki/Exoplanet_naming_convention#Circumbinary_planets_and_2010_proposal
519 733058
>>32990

>а тут-то как?


Наверно будут привязывать к общему центру масс.
520 733070
https://newatlas.com/space/bizarre-star-magnetar-solid-surface-x-ray/

>Bizarre star may have a solid surface, according to X-ray observations


Падажжите епту, разве уже не считалось нормальным что у нейтронок твердая поверхность?
Или это "ученый изнасиловал журналиста" опять?
521 733074
перекатывайте пидары
522 733113
Перекат: >>733086 (OP)
Перекат: >>733086 (OP)
Перекат: >>733086 (OP)
Тред утонул или удален.
Это копия, сохраненная 31 мая 2023 года.

Скачать тред: только с превью, с превью и прикрепленными файлами.
Второй вариант может долго скачиваться. Файлы будут только в живых или недавно утонувших тредах. Подробнее

Если вам полезен архив М.Двача, пожертвуйте на оплату сервера.
« /spc/В начало тредаВеб-версияНастройки
/a//b//mu//s//vg/Все доски